Download as pdf or txt
Download as pdf or txt
You are on page 1of 96

A SERIES

PRESTORMINGTM 2021

TEST 2 -MODERN INDIA I - EXPLANATION

INDEX

QUESTION NO PAGE NO

1. ......................................................................................................................... 4
2. ......................................................................................................................... 4
3. ......................................................................................................................... 6
4. ......................................................................................................................... 7
5. ......................................................................................................................... 7
6. ......................................................................................................................... 9
7. ....................................................................................................................... 10
8. ....................................................................................................................... 11
9. ....................................................................................................................... 12
10. ....................................................................................................................... 13
11. ....................................................................................................................... 14
12. ....................................................................................................................... 15
13. ....................................................................................................................... 16
14. ....................................................................................................................... 17
15. ....................................................................................................................... 18
16. ....................................................................................................................... 20
17. ....................................................................................................................... 21
18. ....................................................................................................................... 22
19. ....................................................................................................................... 23
20. ....................................................................................................................... 24
21. ....................................................................................................................... 25
22. ....................................................................................................................... 26
23. ....................................................................................................................... 27
24. ....................................................................................................................... 28
25. ....................................................................................................................... 29
26. ....................................................................................................................... 30
27. ....................................................................................................................... 31
28. ....................................................................................................................... 32
29. ....................................................................................................................... 32
30. ....................................................................................................................... 33
31. ....................................................................................................................... 35
32. ....................................................................................................................... 36
33. ....................................................................................................................... 37
1
A SERIES

34. ....................................................................................................................... 38
35. ....................................................................................................................... 39
36. ....................................................................................................................... 40
37. ....................................................................................................................... 41
38. ....................................................................................................................... 43
39. ....................................................................................................................... 44
40. ....................................................................................................................... 46
41. ....................................................................................................................... 47
42. ....................................................................................................................... 48
43. ....................................................................................................................... 49
44. ....................................................................................................................... 50
45. ....................................................................................................................... 50
46. ....................................................................................................................... 51
47. ....................................................................................................................... 52
48. ....................................................................................................................... 53
49. ....................................................................................................................... 54
50. ....................................................................................................................... 55
51. ....................................................................................................................... 56
52. ....................................................................................................................... 57
53. ....................................................................................................................... 58
54. ....................................................................................................................... 59
55. ....................................................................................................................... 60
56. ....................................................................................................................... 60
57. ....................................................................................................................... 61
58. ....................................................................................................................... 62
59. ....................................................................................................................... 63
60. ....................................................................................................................... 63
61. ....................................................................................................................... 64
62. ....................................................................................................................... 65
63. ....................................................................................................................... 66
64. ....................................................................................................................... 67
65. ....................................................................................................................... 68
66. ....................................................................................................................... 69
67. ....................................................................................................................... 70
68. ....................................................................................................................... 72
69. ....................................................................................................................... 73
70. ....................................................................................................................... 73
71. ....................................................................................................................... 74
72. ....................................................................................................................... 75

2
A SERIES

73. ....................................................................................................................... 77
74. ....................................................................................................................... 78
75. ....................................................................................................................... 79
76. ....................................................................................................................... 80
78. ....................................................................................................................... 81
79. ....................................................................................................................... 83
80. ....................................................................................................................... 83
81. ....................................................................................................................... 84
82. ....................................................................................................................... 85
83. ....................................................................................................................... 86
84. ....................................................................................................................... 87
85. ....................................................................................................................... 88
86. ....................................................................................................................... 89
87. ....................................................................................................................... 90
88. ....................................................................................................................... 90
89. ....................................................................................................................... 91
90. ....................................................................................................................... 93
91. ....................................................................................................................... 93
92. ....................................................................................................................... 93
93. ....................................................................................................................... 94
94. ....................................................................................................................... 94
95. ....................................................................................................................... 95
96. ....................................................................................................................... 95
97. ....................................................................................................................... 95
98. ....................................................................................................................... 95
99. ....................................................................................................................... 96
100. ....................................................................................................................... 96

3
A SERIES

PRESTORMINGTM 2021
TEST 2 – MODERIN INDIA I EXPLANATION

1. With reference to Advent of Europeans in India, which among the following event happened
earliest?
(a) Serampore Mission was established in the area under the control of Danish East India
Company
(b) Dutch captured English ships in the third Anglo Dutch war
(c) French set up their first factory at Surat
(d) Captain Thomas Best of the English East India Company defeated the Portuguese
EXPLANATION:

Danish- Serampore missionary (1800-1845) is India's first Christian missionary


organisation.williamcarey and his two associates established this mission on 10 January 1800. At
the end of the 18th century, a few more missionaries were sent to Bengal to work in collaboration
with Carey. To avoid expulsion by the English Government, they took shelter at the Danish
settlement at Serampore. So, option (a) is not correct.
In the third Anglo-Dutch War (1672-74), communications between Surat and the new English
settlement of Bombay got cut due to which three homebound English ships were captured in the
Bay of Bengal by the Dutch forces. The retaliation by the English resulted in the defeat of the Dutch,
in the battle of Hooghly (November 1759), which dealt a crushing blow to Dutch ambitions in India.
So, option (b) is not correct.
In 1667, French Set up their first factory at Surat. So, option (c) is not correct.
It was in 1612 that Captain Thomas Best defeated the Portuguese in the sea off Surat; an impressed
Jahangir granted permission to the English in early 1613 to establish a factory
at Surat under Thomas Aldworth. So, option (d) is correct.

ADDITIONAL INFORMATION:

Why a Sea Route to India

 Spirit of renaissance in the 15th-century Europe.


 European economy growing rapidly, leading to prosperity and demand for luxury goods;
increase in the supply of meatrequiring spices for preservation.
 Capture of Constantinople in 1453, and Syria and Egypt later by the Ottoman Turks calling
for a new route to reach India without dealing with Arabs and Turks.
 Venice and Genoa too small to stand up to the Turks.
 Spain and Portugal aided with money and men by the North Europeans and by ships and
technical knowledge by theGenoese.
 The Portuguese the pioneers followed by the Dutch, English,Danes and the French respectively
to reach India.

2. With reference to early Modern History India, which among the following event happened
earliest?
(a) Treaty of Amritsar between the English and Ranjit Singh

4
A SERIES

(b) Appointment of Sir Thomas Munro as governor of Madras


(c) Introduction of Permanent Settlement of land revenue in Bengal
(d) Foundation of Calcutta Medical College
EXPLANATION:

Ranjit Singh agreed to sign the Treaty of Amritsar (April 25, 1809) with the English East
India Company.Ranjit Singh acknowledged the British right over the cis-Sutlej territories.
So, option (a) is not correct.
In 1819, he was appointed governor of Madras, where he founded systems of revenue
assessment and general administration which substantially persisted into the twentieth
century. He is regarded as the father of the 'Ryotwari system'. So, option (b) is not correct.
The Permanent Settlement of Bengal was brought into effect by the East India Company
headed by the Governor-General Lord Cornwallis in 1793. This was basically an agreement
between the company and the Zamindars to fix the land revenue. First enacted in Bengal,
Bihar and Odisha, this was later followed in northern Madras Presidency and the district
of Varanasi. So, option (c) is correct.
Calcutta Medical College, officially Medical College and Hospital, Kolkata, is an Indian
medical school and hospital. The school was established in 1835 by Lord William Bentinck
as Medical College, Bengal during British Raj. So, option (d) is not correct.

ADDITIONAL INFORMATION:

Three major systems of land revenue collection existed in India. They were – Zaminidari, Ryotwari and
Mahalwari.

Zamindari System

 Zamindari System was introduced by Cornwallis in 1793 through Permanent Settlement Act.
It was introduced in provinces of Bengal, Bihar, Orissa and Varanasi.Also known as Permanent
Settlement System.
 Zamindars were recognized as owner of the lands. Zamindars were given the rights to collect
the rent from the peasants.The realized amount would be divided into 11 parts. 1/11 of the
share belongs to Zamindars and 10/11 of the share belongs to East India Company.

Ryotwari System

 Ryotwari System was introduced by Thomas Munro in 1820.


 Major areas of introduction include Madras, Bombay, parts of Assam and Coorgh provinces of
British India.
 In Ryotwari System the ownership rights were handed over to the peasants. British
Government collected taxes directly from the peasants.The revenue rates of Ryotwari System
were 50% where the lands were dry and 60% in irrigated land.

Mahalwari System

 Mahalwari system was introduced in 1833 during the period of William Bentick.
 It was introduced in Central Province, North-West Frontier, Agra, Punjab, Gangetic Valley, etc
of British India.

5
A SERIES

 The Mahalwari system had many provisions of both the Zamindari System and Ryotwari
System. In this system, the land was divided into Mahals. Each Mahal comprises one or more
villages.Ownership rights were vested with the peasants.
 The villages committee was held responsible for collection of the taxes.

3. Consider the following statements:


1. The police force was very organized during Governor-Generalship of William Bentinck
2. Warren Hastings abolished the institution of the faujdar and amils
3. Cornwallis relieved the zamindars of their police duties.
Which of the statements given above is/are correct?
(a) 3 only
(b) 1 and 2 only
(c) 2 and 3 only
(d) 1, 2 and 3
EXPLANATION:

Bentinck (governor-general, 1828-35) abolished the office of the SP. The collector/magistrate was
now to head the police force in his jurisdiction and the commissioner in each division was to act as
the SP. This arrangement resulted in a badly organised police force, putting a heavy burden on the
collector/magistrate. So, statement 1 is not correct.
In 1774, Warren Hastings restored the institution of faujdars and asked the zamindars to assist
them in suppression of dacoits, violence and disorder. So, statement 2 is not correct.
1791 Cornwallis organised a regular police force to maintain law and order by going back to and
modernising the old Indian system of thanas (circles) in a district under a daroga (an Indian) and a
superintendent of police (SP) at the head of a district. He relieved the zamindars of their police
duties. So, statement 3 is correct.

ADDITIONAL INFORMATION:

 Under the Mughal rule there were the faujdars who helped in maintaining law and order, and
amils who were basically revenue collectors but had to contend with rebels, if any. The kotwal
was responsible for maintenance of law and order in the cities.
 In 1770, the institution of the faujdar and amils were abolished. However, in 1774, Warren
Hastings restored the institution of faujdars and asked the zamindars to assist them in
suppression of dacoits,violence and disorder.
 In 1775, faujdarthanas were established in the major towns of large districts and were assisted
by several smaller police stations.
 The recommendations of the Police Commission (1860) led to the Indian Police Act, 1861. The
commissionrecommended—
o a system of civil constabulary—maintaining the village set-up in the present form (a
village watchman maintained by the village) but in direct relationship with the rest of
the constabulary.
o inspector-general as the head in a province, deputyinspector-general as the head in
range, and SP asthe head in a district.
 The police gradually succeeded in curbing criminal acts, such as dacoity, thugee, etc. But,
while dealing with the public, the attitude of the police was unsympathetic. The police was also
used to suppress the national movement.
6
A SERIES

 The British did not create an All-India Police. The Police Act, 1861 presented the guidelines for
a police setup in the provinces. The ranks were uniformly introduced allover the country.
 1902 The Police Commission recommended the establishment of CID (Criminal Investigation
Department) inthe provinces and a Central Intelligence Bureau at the Centre.

4. Consider the following statements with reference to the early development of Judiciary in
British India:
1. Cornwallis setup the District Fauzdari Adalats and District Diwani Adalats to try criminal
disputes and civil disputes respectively.
2. William Bentinck abolished the four Circuit Courts and transferred their functions to
collectors.
Which of the statements given above is/are correct?
(a) 1 only
(b) 2 only
(c) Both 1 and 2
(d) Neither 1 nor 2
EXPLANATION:

Warren Hastings (1772-1785) setup the District Fauzdari Adalats and District Diwani Adalats to try
criminal disputes and civil disputes respectively. District Diwaniadalats were placed under the
collector and had Hindu law applicable for Hindus and the Muslim law for Muslims.District Fauzdari
Adalats were placed under an Indian officer assisted by qazis and muftis. These adalats also were
under the general supervision of the collector. Muslim law was administered in Fauzdari Adalats.
So, statement 1 is not correct.
William Bentinck (1828-1833) abolished the four Circuit Courts and their functions transferred to
collectors under the supervision of the commissioner of revenue and circuit. So, statement 2 is
correct.

ADDITIONAL INFORMATION:

Reforms under William Bentinck (1828-1833)

 The four Circuit Courts were abolished and their functions transferred to collectors under the
supervision ofthe commissioner of revenue and circuit.
 SadarDiwani Adalat and a Sadar Nizamat Adalat were set up at Allahabad for the convenience
of the people ofUpper Provinces.
 Till now, Persian was the official language in courts. Now, the suitor had the option to use
Persian or a vernacular language, while in the Supreme Court, English language replaced
Persian.
 1833 : A Law Commission was set up under Macaulay for codification of Indian laws. As a
result, a Civil ProcedureCode (1859), an Indian Penal Code (1860) and a Criminal Procedure
Code (1861) were prepared.

5. Due to which one of the following reasons, there was cheap and machine-made imports
flooded the Indian market in the early nineteenth century in India?
(a) Indian people were more interested in the western culture and thoughts
(b) The Charter Act of 1813 allowed one-way free trade for the British citizens

7
A SERIES

(c) Deindustrialisation ruined the production of Indian Artisans and Handicraftsmen


(d) The emergence of the commercialisation of agriculture
EXPLANATION:

Westernisation of India is not one of the reasons for cheap and machine-made imports flooded the
Indian market. So, option (a) is not correct.
Cheap and machine-made imports flooded the Indian market after the Charter Act of 1813 allowing
one-way free trade for the British citizens. On the other hand, Indian products found it more and
more difficult to penetrate the European markets. Tariffs of nearly 80 per cent were imposed on
Indian textiles so that Indian cloth could no longer be cheap. The newly introduced rail network
helped the European products to reach the remotest corners of the country. From being a net
exporter, India became a net importer. So, option (b) is correct.
Many artisans, faced with diminishing returns and repressive policies, abandoned their professions,
moved to villages and took to agriculture. This resulted in increased pressure on land. An
overburdened agriculture sector was a major cause of poverty during British rule and this upset the
village economic set-up. It was not the reason for cheap and machine-made imports flooded the
Indian market. So, option (c) is not correct.
The emergence of the commercialisation of agriculture in India. For the Indian peasant,
commercialisation seemed a forced process. There was hardly any surplus for him to invest in
commercial crops, given the subsistence level at which he lived, while commercialisation linked
Indian agriculture with international market trends and their fluctuations. So, option (d) is not
correct.

ADDITIONAL INFORMATION:

Economic Impact of British Rule

 Deindustrialisation—ruin of artisans and handicraftsmen.


 Impoverishment of peasantry—ruralisation of India.
 Emergence of new land relations—ruin of old zamindars.
 Stagnation and deterioration of agriculture.
 Commercialisation of Indian agriculture.
 Development of modern industry.
 Rise of Indian national bourgeoisie.
 Economic drain.
 Famine and poverty

Nationalist Critique

 India getting poorer due to colonial exploitation.


 Problem of poverty—a national problem of raising productivecapacities and energy.
 Development equated with industrialisation, which should takeplace through Indian, not
foreign capital.
 British policies on trade, finance, infrastructure development,expenditure designed to serve
imperialist interests.
 Need for complete severance of India’s economic subservienceto Britain and development of an
independent economy.

8
A SERIES

6. Lord Wellesley enacted Censorship of Press Act in the late eighteenth century mainly due
to
(a) The newspapers reached London and exposed their misdeeds
(b) The anticipation of French invasion to India
(c) Ban the Indian language newspapers and those edited by Indians
(d) Curb the education, formation and propagation of nationalist ideology
EXPLANATION:

Lord Wellesley enacted this, anticipating French invasion of India. It imposed almost
wartime press restrictions including pre-censorship. This act brought all the newspapers
under the Government scrutiny before their publication. This act was later extended in
1807 and covered all kinds of Press Publications newspapers, magazine, books and
Pamphlets. These restrictions were relaxed under Lord Hastings, who had progressive
views, and in 1818, pre-censorship was dispensed with. So, option (b) is correct.

ADDITIONAL INFORMATION:

Early Regulations of Indian press

 Censorship of Press Act, 1799

Licensing Regulations, 1823

 The acting governor-general, John Adams, who had reactionary views, enacted these.
According to these regulations, startingor using a press without licence was a penal offence.
 Later on, the Act was extended to cover journals, pamphlets and books. These restrictions were
directed chiefly against Indian language newspapers or those edited by Indians.
RammohanRoy’s Mirat-ul-Akbar had to stop publication.

Press Act of 1835 or Metcalfe Act

 Metcalfe (governor-general—1835-36) repealed the obnoxious 1823 ordinance and earned the
epithet, “liberator of the Indian press”. The new Press Act (1835) required a printer/publisher
to give a precise account of premises of a publication and cease functioning, if required by a
similardeclaration.
 The result of a liberal press policy was a rapid growthof newspapers.

Licensing Act, 1857

 Due to the emergency caused by the 1857 revolt, this Act imposed licensing restrictions in
addition to the already existing registration procedure laid down by Metcalfe Act and the
government reserved the right to stop publication andcirculation of any book, newspaper or
printed matter.

9
A SERIES

Registration Act, 1867

 This replaced Metcalfe’s Act of 1835 and was of a regulatory, not restrictive, nature. As per the
Act, (i) every book/ newspaper was required to print the name of the printer and the publisher
and the place of the publication; and (ii) a copy was to be submitted to the local government
within onemonth of the publication of a book.

7. Consider the following pairs:


(News paper) (Founder)
1. Banga-Duta - Dadabhai Naoroji
2. East Indian - Henry Vivian Derozio
3. Hindoo Patriot - Harishchandra Mukerji
Which of the pairs given above is/are correctly matched?
(a) 1 only
(b) 2 and 3 only
(c) 1 and 2 only
(d) 1, 2 and 3
EXPLANATION:

Bangaduta (1822) is a weekly in 4 languages (English,Bengali, Persian,Hindi) founded by


Rajaram Mohan Roy, Dwarakanath Tagore and others. So, pair 1 is not correct.
Derozio, Henry (1809-1831) poet, rationalist thinker and teacher. Henry Louis Vivian
Derozio was appointed a teacher of the Calcutta hindu college in May 1826 when he was
only seventeen. Derozio was also actively involved in promoting the welfare of his Eurasian
community, and had begun editing a daily English newspaper, The East Indian. So, pair
2 is correct.
Harish Chandra Mukherjee was an Indian journalist and patriot, who fought for theindigo
cultivators (and against the indigo planters) and forced the government to bring about
changes. He was associated with the Hindu Patriot right form its start in 1853. In 1855,
he secured the ownership and became the editor of the newspaper. So, pair 3 is correct.

ADDITIONAL INFORMATION:

Young Bengal Movement and Henry Vivian Derozio

 During the late 1820s and early 1830s, there emerged a radical, intellectual trend among the
youth in Bengal, whichcame to be known as the ‘Young Bengal Movement’.
 A young Anglo-Indian, Henry Vivian Derozio (1809-31), who taught at the Hindu College from
1826 to 1831, was the leader and inspirer of this progressive trend.
 Drawing inspiration from the great French Revolution, Derozio inspired his pupils to think
freely and rationally, question all authority, love liberty, equality and freedom, and oppose
decadent customs and traditions.
 The Derozians also supported women’s rights and education. Also, Derozio was perhaps the
first nationalist poet of modern India.
 The Derozians, however, failed to have a long-term impact. Derozio was removed from the
Hindu College in 1831 because of his radicalism.

10
A SERIES

 The main reason for their limited success was the prevailing social conditions at that time,
which were not ripe for the adoption of radical ideas.
 Further, there was no support from any other social group or class. The Derozians lacked any
real link with the masses; for instance, they failed to take up the peasants’ cause. In fact, their
radicalism was bookish in character.
 But, despite their limitations, the Derozians carried forward Rammohan Roy’s tradition of
public education on social, economic and political questions. For instance, they demanded
induction of Indians in higher grades of services, protection of ryots from oppressive zamindars,
better treatment to Indian labour abroad in British colonies, revision of the Company’s charter,
freedom of press and trial by jury.

8. Which of the following statements is/are not correct with reference to the Wood’s Dispatch
of 1854?
(a) It did not mention about female and vocational education
(b) It laid down that the education imparted in government institutions should be secular
(c) It made English as the medium of instruction for higher studies
(d) It mandated to setup Education Department in every province

EXPLANATION:

In 1854, Charles Wood prepared a despatch on an educational system for India. Considered
the “Magna Carta of English Education in India”, this document was the first
comprehensive plan for the spread of education in India.
 It asked the government of India to assume responsibility for education of the
masses, thus repudiating the ‘downward filtration theory’, at least on paper.
 It systematised the hierarchy from vernacular primary schools in villages at bottom,
followed by Anglo-Vernacular High Schools and an affiliated college at the district
level, and affiliating universities in the presidency towns of Calcutta, Bombay and
Madras.
 It recommended English as the medium of instruction for higher studies and
vernaculars at school level.
 It laid stress on female and vocational education, and on teachers’ training.
 It laid down that the education imparted in government institutions should be
secular.
 It recommended a system of grants-in-aid to encourage private enterprise.
So, option (a) is correct.

ADDITIONAL INFORMATION:

Developments after Wood’s Despatch of 1854

 In 1857, universities at Calcutta, Bombay and Madras were set up and later, departments of
education were set up in allprovinces.
 The Bethune School founded by J.E.D. Bethune at Calcutta (1849) was the first fruit of a
powerful movement for education of women which arose in 1840s and 1850s.Bethune was the
president of the Council of Education.

11
A SERIES

 Mostly due to Bethune’s efforts, girls’ schools were set up on a sound footing and brought
under government’s grantsin-aid and inspection system.
 An Agriculture Institute at Pusa (Bihar) and an Engineering Institute at Roorkee were started.
 The ideals and methods of Wood’s Despatch dominated the field for five decades which saw
rapid westernisation of education system in India, with educational institutions run by
European headmasters and principals. Missionary enterprises played their own part.
Gradually, private Indianeffort appeared in the field.

9. With reference to development of Vernacular Education in India, who among the following
developed a comprehensive scheme of village education through the medium of vernacular
languages in Northwest Provinces?
(a) William Adam
(b) James Jonathan
(c) Lord Dalhousie
(d) Lord Macaulay
EXPLANATION:

William Adam’s reports on vernacular education in Bengal and Bihar pointed out defects
in the system of vernacular education. So, option (a) is not correct.
James Thomson, lieutenant-governor of Northwest Provinces (1843- 53), developed a
comprehensive scheme of village education through the medium of vernacular languages.
In these village schools, useful subjects such as mensuration and agriculture sciences were
taught. The purpose was to train personnel for the newly set up Revenue and Public Works
Department. So, option (b) is correct.
In 1853, Dalhousie passed several regulations concerning the primary education. In 1854,
he received instructions from the president of the Board of Control, Sir Charles Wood,
concerning education in India. It was suggested that schools and colleges should be opened
in every district and a university should be established in each Presidency town, i.e,
Bombay, Madras and Calcutta. So, option (c) is not correct.
The famous Lord Macaulay’s Minute settled the row in favour of Anglicists—the limited
government resources were to be devoted to teaching of Western sciences and literature
through the medium of English language alone. Lord Macaulay held the view that “Indian
learning was inferior to European learning”—which was true as far as physical and social
sciences in the contemporary stage were concerned. So, option (d) is not correct.

ADDITIONAL INFORMATION:

Lord Macaulay’s Minute (1835)

 The famous Lord Macaulay’s Minute settled the row in favour of Anglicists—the limited
government resources were to be devoted to teaching of Western sciences and literaturethrough
the medium of English language alone.
 Lord Macaulay held the view that “Indian learning was inferior to European learning”—which
was true as far as physical and social sciences in the contemporary stage were concerned.
 The government soon made English as the medium of instruction in its schools and colleges
and opened a few English schools and colleges instead of a large number of elementary schools,
thus neglecting mass education.

12
A SERIES

 The British planned to educate a small section of upper and middle classes, thus creating a
class “Indian in blood and colour but English in tastes, in opinions, in morals and in intellect”
who would act as interpreters between the government and masses and would enrich the
vernaculars by which knowledge of Western sciences and literature would reach the masses.
This was called the ‘downward filtration theory’.
 Modern ideas, if not education, did filter down to the masses, though not in a form desired by
the rulers, but through political parties, press, pamphlets, public platforms, etc.
 Modern education only helped this process by making available the basic literature on physical
and social sciences to nationalists, thus stimulating their capacity to make social analysis—
otherwise the content, structure and curricula of modern education served colonial interests.

10. With reference to the Subsidiary Alliance of Wellesley, consider the following statements:
1. English agreed to help a native ruler with a fixed force in return for a fixed annual amount of
money.
2. An English resident was kept at the court of the native ruler
3. The English agreed not to interfere in internal affairs of the native rulers
Which of the statements given above is/are correct?
(a) 3 only
(b) 1 and 2 only
(c) 2 and 3 only
(d) 1, 2 and 3

EXPLANATION:

Terms of subsidiary alliance


 An Indian ruler entering into a subsidiary alliance with the British would accept
British forces within his territory and to pay for their maintenance. So, statement
1 is correct.
 The ruler would accept a British official (resident) in his state. So, statement 2 is
correct.
 The ruler who entered into a subsidiary alliance would not join any alliance with
any other power or declare war against any power without the permission of the
British.
 The ruler would dismiss any Europeans other than the British and avoid employing
new ones.
 The ruler would let the British rule on any conflict with any other state.
 The ruler would acknowledge the East India Company as the paramount power in
India.
 The ruler would have his state be protected by the Company from external dangers
and internal disorders.
 If the rulers failed to make the payments that were required by the alliance, part of
their territory would be taken away as a penalty.
In return for all this, the British would defend the ruler from his enemies and adopt a policy
of non-interference in the internal matters of the allied state. So, statement 3 is correct.

13
A SERIES

ADDITIONAL INFORMATION:

Subsidiary Alliance

 The subsidiary alliance system was used by Lord Wellesley,who was governor-general from
1798-1805, to build anempire in India.
 Subsidiary alliances brought immense gains for the East India Company by extending the
areas under British control and bringing relative peace in subsidies and/or territory.
 During the seven-year rule of Wellesley alone, over 100small and big states of India signed the
subsidiary treaty.
 It was probably Dupleix, who first gave on hire (so to say) European troops to Indian rulers to
fight their wars.
 Since then, almost all the governor-generals from Clive onwards applied the system to various
Indian states and brought it tonear perfection.
 The first Indian state to fall into this protection trap (which anticipated the subsidiary alliance
system) was Awadh which in 1765 signed a treaty under which the Company pledged to defend
the frontiers of Awadh on the condition of the Nawab defraying the expenses of such defence.
 It was Wellesley’s genius to make it a general rule to negotiate for the surrender of territory in
fullsovereignty for the maintenance of the subsidiary force.

11. Consider the following statements regarding the recent World Economic Outlook released
by the International Monetary Fund.
1. India has overtaken France and the UK to become the world’s fifth-largest economy.
2. India’s performance is estimated gross domestic product based on purchasing power parity,
abbreviated GDP (PPP).
Which of the statements given above is/are correct?

(a) 1 only
(b) 2 only
(c) Both 1 and 2
(d) Neither 1 Nor 2

EXPLANATION:

India's economy is the fifth largest in the world with a GDP of $ 2.94 trillion, overtaking
the UK and France in 2019 to take the fifth spot. The size of the UK economy is $ 2.83
trillion and that of France is $ 2.71 trillion. So, statement 1 is correct.

India’s performance is estimated based on the nominal gross domestic product (GDP) not
based on purchasing power parity (PPP). When ranked by nominal GDP, the country
leapfrogged France and the UK. So, statement 2 is not correct.

ADDITIONAL INFORMATION:

About the World Economic Outlook:

 The World Economic Outlook (WEO) is an IMF report that provides analysis and forecasts of
economic developments and policies in its member countries.

14
A SERIES

 The report projects the state of the global economy and highlights risks and uncertainty that
threaten growth.
 It also projects developments in the global financial markets and economic systems.
 The WEO is usually prepared twice a year and is used in meetings of the International Monetary
and Financial Committee.

12. Who among the following completely destroyed the systematised plunder of Pindaris?
(a) Sir George Barlow
(b) William Bentinck
(c) Francis Rawdon-Hastings
(d) Richard Wellesley
EXPLANATION:

Sir George Barlow (1805 – 1807) was served as Acting Governor-General of India from the
death of Lord Cornwallis in 1805 until the arrival of Lord Minto in 1807. He was famous
for Mutiny of Vellore in 1806 in which the Indian soldiers killed many English officials. So,
option (a) is not correct.
Lord William Bentinck served as Governor General of India between 1828 to 1835. His
tenure is known for the social reforms such as Abolition of Sati in 1829, Suppression of
Thugi, and Suppression of Infanticide etc. English was introduced as a medium of higher
education on the advice of his council member, Thomas Babington Macaulay. So, option
(b) is not correct.
The Pindaris, made up of many castes and classes, were attached to Maratha armies as
mercenaries. When the Marathas became weak, the Pindaris could not get regular
employment. As a consequence, they started plundering neighbouring territories, including
those of the Company. The English charged the Marathas with giving shelter to the
Pindaris. Pindari leaders like Amir Khan and Karim Khan surrendered while Chitu Khan
fled into the jungles.Lord Hastings’ actions taken against the Pindaris were seen as a
transgression of the sovereignty of the Marathas. So, option (c) is correct.
Richard Colley Wellesley (1798 – 1805) Called himself “Bengal Tiger” and reversed the Non
intervention policy of John Shore. He Opened Fort William College at Calcutta to train the
Company’s servants in Indian languages & customs; but it was closed in 1802. He
formulated the Subsidiary alliance to remove French & establish British Power. So, option
(d) is not correct.

ADDITIONAL INFORMATION:

 Lord Hastings had the imperialistic design of imposing British paramountcy. By the Charter
Act of 1813, the East India Company’s monopoly of trade in China (except tea)ended and hence
the company needed more markets.
 The Pindaris, made up of many castes and classes, were attached to Maratha armies as
mercenaries.
 When the Marathas became weak, the Pindaris could not get regular employment. As a
consequence, they started plundering neighbouringterritories, including those of the Company.
 The English charged the Marathas with giving shelter to the Pindaris. Pindari leaders like Amir
Khan and Karim Khan surrenderedwhile Chitu Khan fled into the jungles.

15
A SERIES

 The Treaty of Bassein, described as “a treaty with a cipher (the Peshwa)”, wounded the feelings
of the other Maratha leaders. They saw the treaty as an absolute surrender of independence.
 Lord Hastings’ actions taken against the Pindaris were seen as a transgression of the
sovereignty of the Marathas;they served to once again unite the Maratha confederacy.
 A repentant Bajirao II made a last bid in 1817 by rallying together the Maratha chiefs against
the English in course ofthe Third Anglo-Maratha War.

13. Consider the following statements regarding the Pradhan MantriKisanSampada Yojana.
1. The objective of PMKSY is to supplement agriculture, modernize processing and decrease Agri-
Waste.
2. It is an umbrella scheme incorporating the ongoing schemes such as Mega Food Parks,
Integrated Cold Chain and Value Addition Infrastructure etc.
3. It is being implemented jointly by the Ministry Agriculture along with the Ministry of Consumer
Affairs, Food and Public Distribution.
Which of the statements given above is/are correct?

(a) 1 only
(b) 1 and 2 only
(c) 2 and 3 only
(d) 1, 2 and 3
EXPLANATION:

The Central Sector Scheme - SAMPADA (Scheme for Agro-Marine Processing and
Development of Agro-Processing Clusters) was approved by the cabinet in May 2017 for the
period of 2016-20 coterminous with the 14th Finance Commission cycle. The scheme has
now been renamed as the "Pradhan MantriKisanSampada Yojana (PMKSY)". The objective
of PMKSY is to supplement agriculture, modernize processing and decrease Agri-Waste.
So, statement 1 is correct.

It is an umbrella scheme incorporating ongoing schemes of the Ministry like Mega Food
Parks, Integrated Cold Chain and Value Addition Infrastructure, Food Safety and Quality
Assurance Infrastructure, etc. and also new schemes like Infrastructure for Agro-
processing Clusters, Creation of Backward and Forward Linkages, Creation / Expansion
of Food Processing & Preservation Capacities. So, statement 2 is correct.

It is being implemented jointly by the Ministry of Food Processing Industries. So,


statement 3 is not correct.

ADDITIONAL INFORMATION:

Under PMKSY the following schemes are to be implemented.

 Mega Food Parks


 Integrated Cold Chain, Value Addition and Preservation Infrastructure
 Creation/Expansion of Food Processing/Preservation Capacities
 Infrastructure for Agro Processing Clusters
 Scheme for Creation of Backward and Forward Linkages
16
A SERIES

 Food Safety & Quality Assurance Infrastructure


 Human Resources and Institutions

Impact

 The implementation of PMKSY will result in creation of modern infrastructure with efficient
supply chain management from farm gate to retail outlet.
 It will provide a big boost to the growth of food processing sector in the country.
 It will help in providing better prices to farmers and is a big step towards doubling of farmers’
income.
 It will create huge employment opportunities especially in the rural areas.
 It will also help in reducing wastage of agricultural produce, increasing the processing level,
availability of safe and convenient processed foods at affordable price to consumers and
enhancing the export of the processed foods.

14. Recently which of the following regions have been identified as Algal Bloom hotspots under
the Algal Bloom Information Service?
1. North Eastern Arabian Sea
2. Gulf of Mannar
3. Coastal waters off Kerala
4. Coastal waters of Gopalpur.
Select the correct answer using the code given below.

(a) 1 and 2 only


(b) 1, 2 and 3 only
(c) 3 and 4 only
(d) 1, 2, 3 and 4
EXPLANATION:

Algal Bloom Information Service (ABIS) provides timely information on harmful algal
blooms, which are detrimental to coastal fisheries, water quality and also tend to induce
respiratory problems within the coastal population from time to time. The Indian National
Centre for Ocean Information Services (INCOIS)-ABIS will provide near-real time
information on the Spatio-temporal occurrence and spread of phytoplankton blooms over
the North Indian Ocean.

Four regions have been identified as bloom hotspots

 North Eastern Arabian Sea


 coastal waters off Kerala
 Gulf of Mannar
 coastal waters of Gopalpur.

So, option (d) is correct.

17
A SERIES

ADDITIONAL INFORMATION:

 The Indian National Centre for Ocean Information Services (Incois), Hyderabad, has launched
three ocean-based specialised products/services — the Small Vessel Advisory and Forecast
Services System (SVAS), the Swell Surge Forecast System (SSFS) and the Algal Bloom
Information Service (ABIS).
 Incois, an autonomous organisation under the Ministry of Earth Sciences, provides a number
of free services for users in the marine realm, including fishermen.
 While SVAS is aimed at improving the operations of numerous small marine vessels,
particularly fishing vessels, the SSFS forewarns the coastal population about swell waves. The
coastal region experiences frequent damages caused by swell waves that originate from the
distant Southern Indian Ocean.
 The ABIS provides information on harmful algal blooms that are detrimental to coastal fisheries
and also tend to cause respiratory problems in the coastal population.

Advisory service

 The SVAS is an innovative impact-based advisory and forecast service system for small vessels
operating in Indian coastal waters, said an Incois release. It warns users about potential zones
where vessel overturning can take place, 10 days in advance. The advisories are valid for small
vessels of beam width up to 7 m. This limit covers the fishing vessels used in all the nine
coastal States and Union Territories.
 The warning system is based on the Boat Safety Index (BSI) derived from wave model forecast
outputs such as significant wave height, wave steepness, directional spread and the rapid
development of wind at sea, which is boat-specific.

The SSFS is designed to predict the kallakkadal, or swell surge, that occurs along the Indian coast,
particularly the West. These are flashflood events that take place without any noticeable advance
change in local winds or any other apparent signature in the coastal environment. Kallakkadal is
a colloquial term used by Kerala fishermen to refer to the ‘freaky’ flooding episodes. Since most
people mistake these for tsunamis, the SSFS will be useful, said the release.
15. Consider the following statements regarding the Law Commission of India.
1. It is a statutory body works under the Ministry of Law and Justice.
2. Its recommendations are necessary when judges are appointed at the High Court.
3. The Commission is reconstituted for every 5 years.
Which of the statements given above is/are notcorrect?

(a) 1 only
(b) 1 and 2 only
(c) 2 and 3 only
(d) 1, 2 and 3

EXPLANATION:

Law Commission of India is an executive body established by an order of the Government


of India. Its major function is to work for legal reform. Its membership primarily comprises
legal experts, who are entrusted a mandate by the Government. The commission is

18
A SERIES

established for a fixed tenure and works as an advisory body to the Ministry of Law and
Justice. So, statement 1 is not correct.

The recommendations of the commission are not binding on the government. They may be
accepted or rejected. Action on the said recommendations depends on the
ministries/departments, which are concerned with the subject matter of the
recommendations. So, statement 2 is not correct.

Originally formed in 1955, the commission is reconstituted every three years and so far,
277 reports have been submitted to the government. So, statement 3 is not correct.

ADDITIONAL INFORMATION:

The 22nd Law Commission will be constituted for a period of three years from the date of publication
of its Order in the Official Gazette. It will consist of:

 a full-time Chairperson;
 four full-time Members (including Member-Secretary)
 Secretary, Department of Legal Affairs as ex-officio Member;
 Secretary, Legislative Department as ex officio Member; and
 not more than five part-time Members.

Terms of reference

 The Law Commission shall, on a reference made to it by the Central Government or suo-motu,
undertake research in law and review of existing laws in India for making reforms therein and
enacting new legislations.
 It shall also undertake studies and research for bringing reforms in the justice delivery systems
for elimination of delay in procedures, speedy disposal of cases, reduction in cost of litigation
etc.

The Law Commission of India shall, inter-alia

 identify laws which are no longer needed or relevant and can be immediately repealed
 examine the existing laws in the light of DPSP and Preamble
 consider and convey to the Government its views on any subject relating to law and judicial
administration that may be specifically referred to it by the Government through Ministry of
Law and Justice (Department of Legal Affairs);
 Consider the requests for providing research to any foreign countries as may be referred to it
by the Government through the Ministry of Law and Justice (Department of Legal Affairs);
 take all such measures as may be necessary to harness law and the legal process in the service
of the poor;
 revise the Central Acts of general importance so as to simplify them and remove anomalies,
ambiguities and inequities;

19
A SERIES

16. Consider the following statements regarding the National Rurban Mission
1. It follows the vision of Development of a cluster of villages that preserve and nurture the
essence of rural community life.
2. It is applicable only for the Tribal States scheduled under the Indian Constitution.
Which of the statements given above is/are correct?

(a) 1 only
(b) 2 only
(c) Both 1 and 2
(d) Neither 1 Nor 2
EXPLANATION:

The National Rurban Mission (NRuM) follows the vision of "Development of a cluster of
villages that preserve and nurture the essence of rural community life with focus on equity
and inclusiveness without compromising with the facilities perceived to be essentially
urban in nature, thus creating a cluster of "Rurban Villages". So, statement 1 is correct.

There will be two categories of clusters under NRuM: Non-Tribal and Tribal The process of
selection will vary for each of these categories. While selecting the Rurban cluster the State
may identify a large village/gram panchayat that are growth centers with resources
available in the area that could potentially lead the economic transformation of the region.
Hence, it is not applicable only for the Tribal States scheduled under the Indian
Constitution. So, statement 2 is not correct.

ADDITIONAL INFORMATION:

National Rurban Mission (NRuM)

 The aim of SPMRM is to create 300 rural growth clusters across the country. In the first
phase, 100 clusters will be taken up, thereafter more clusters will be identified based on the
progress of the scheme.
 The objective of the National Rurban Mission (NRuM) is to stimulate local economic
development, enhance basic services, and create well planned Rurban clusters.
 The larger outcomes envisaged under this Mission are:
o Bridging the rural-urban divide-viz: economic, technological and those related to
facilities and services.
o Stimulating local economic development with emphasis on reduction of poverty and
unemployment in rural areas.
o Spreading development in the region.
o Attracting investment in rural areas.

Rurban Cluster

 A 'Rurban cluster', would be a cluster of geographically contiguous villages with a population


of about 25000 to 50000 in plain and coastal areas and with a population of 5000 to 15000 in
desert, hilly or tribal areas. As far as practicable, clusters of villages would follow administrative

20
A SERIES

convergence units of Gram Panchayats and shall be within a single block/tehsil for
administrative convenience.

17. The famous “Pledge Movement” to inspire people to take an oath to prohibit child marriage
was launched by
(a) Brahma Samaj
(b) Indian Social Conference
(c) PrarthanaSamaj
(d) Arya Samaj
EXPLANATION:

BrahmoSamaj is the societal component of Brahmoism, which began as a monotheistic


reformist movement of the Hindu religion that appeared during the Bengal Renaissance. It
was started at Calcutta on 20 August 1828 by Raja Ram Mohan Roy and Debendranath
Tagore. So, option (a) is not correct.
Indian Social Conference was founded by M.G. Ranade and Raghunath Rao, the Indian
Social Conference met annually from its first session in Madras in 1887 at the same time
and venue as the Indian National Congress. It focussed attention on the social issues of
importance; it could be called the social reform cell of the Indian National Congress, in fact.
The conference advocated inter-caste marriages, opposed polygamy and kulinism. It
launched the ‘Pledge Movement’ to inspire people to take a pledge against child marriage.
So, option (b) is correct.
PrarthanaSamaj or "Prayer Society" in Sanskrit, was a movement for religious and social
reform in Bombay, India, based on earlier reform movements. PrarthanaSamaj was
founded by the DadobaPandurang and his brother AtmaramPandurang in 1867 when
Keshub Chandra Sen visited Maharashtra, with an aim to make people believe in one God
and worship only one God. So, option (c) is not correct.
Arya Samaj is a monotheistic Indian Hindu reform movement that promotes values and
practices based on the belief in the infallible authority of the Vedas. The samaj was founded
by Maharishi DayanandSaraswati on 10 April 1875. Members of the Arya Samaj believe in
one God and reject the worship of idols. So, option (d) is not correct.

ADDITIONAL INFORMATION:

Dayananda Saraswati and Arya Samaj

 The Arya Samaj Movement, revivalist in form though not in content, was the result of a reaction
to Western influences.
 Its founder, Dayananda Saraswati or Mulshankar (1824-1883)was born in the old Morvi state
in Gujarat in a brahmin family.
 He wandered as an ascetic for fifteen years (1845-60) in search of truth. The first Arya Samaj
unit was formally set up by him at Bombay in 1875 and later the headquarters ofthe Samaj
were established at Lahore.
 Dayananda’s views were published in his famous work, Satyarth Prakash (The True
Exposition). His vision of India included a classless and casteless society, a united India

21
A SERIES

(religiously, socially and nationally), and an India free from foreign rule, with Aryan religion
being the common religion of all.
 He took inspiration from the Vedas and considered them to be ‘India’s Rock of Ages’, the
infallible and the true original seed of Hinduism. He gave the slogan “Back to theVedas”.
 Dayananda launched a frontal attack on Hindu orthodoxy, caste rigidities, untouchability,
idolatry, polytheism, belief in magic, charms and animal sacrifices, taboo on sea
voyages,feeding the dead through shraddhas, etc.

18. With reference to Lord Cornwallis, Consider the following statements


1. He reformed the Board of Trade which managed the commercial investments of the Company
2. He prepared the law code with the help of Sir John Shore
3. He appointed Indian judges to all the courts at the bottom of the judicial system
Which of the statements given above is/are correct?
(a) 2 only
(b) 1 and 2 only
(c) 1 and 3 only
(d) 1, 2 and 3
EXPLANATION:

Cornwallis reformed the Board of Trade which managed the commercial investments of the
Company. With the aid of Charles Grant, he eradicated numerous abuses and corrupt
practices. Fair treatment was given to weavers and Indian workers. He increased the
remuneration for honest service. So, statement 1 is correct.
Cornwallis was better known as a law giver than as an administrator. With the help of his
colleague, George Barlow, Cornwallis prepared a comprehensive code, covering the whole
field of administration’, judicial, police, commercial and fiscal. So, statement 2 is not
correct.
In the work of judicial reorganization, Cornwallis got the services of Sir William Jones, who
was a judge and a great scholar. Civil and criminal courts were completely reorganized.
Indian judges or Munsiffs were appointed to all the courts at the bottom of the judicial
system. So, statement 3 is correct.

ADDITIONAL INFORMATION:

Reforms under Cornwallis (1786-1793)— Separation of Powers

 The District Fauzdari Courts were abolished and, instead, circuit courts were established at
Calcutta, Dacca, Murshidabad and Patna. These circuit courts had European judges and were
to act as courts of appeal for both civil and criminal cases.
 The Sadar Nizamat Adalat was shifted to Calcutta and was put under the governor-general and
members of the Supreme Council assisted by the chief qazi and the chief mufti.
 The District Diwani Adalat was now designated as the District, City or the Zila Court and placed
under a district judge. The collector was now responsible only for therevenue administration
with no magisterial functions.
 A gradation of civil courts was established (for bothHindu and Muslim laws)—
o Munsiff’s Court under Indian officers,
o Registrar’s Court under a European judge,

22
A SERIES

o District Court under the district judge,


o Four Circuit Courts as provincial courts of appeal,
o SadarDiwani Adalat at Calcutta, and
o King-in-Council for appeals of 5000 pounds and above.
 The Cornwallis Code was laid out—
o There was a separation of revenue and justice administration.
o European subjects were also brought under jurisdiction.
o Government officials were answerable to the civil courts for actions done in their official
capacity.
o The principle of sovereignty of law was established

19. Which of the following is/are the significance of Vellore Mutiny of 1806?
1. The new dress regulations were abolished
2. Flogging for Indian soldiers was abolished
3. All three Madras regiments involved in the Vellore Mutiny were disbanded
Select the correct answer from the code given below.
(a) 2 only
(b) 2 and 3 only
(c) 3 only
(d) 1, 2 and 3
EXPLANATION:

The major impact of the Vellore mutiny are given below:


 All three Madras regiments involved in the Vellore Mutiny were disbanded. So,
statement 3 is correct.
 After a trial, sepoys involved in the mutiny were punished by death (blown away
from canons, hanging and firing squads) and by penal transportation.
 John Craddock and other senior British officers responsible for the new dress
regulations were recalled to Britain.
 The new dress regulations were abolished. So, statement 1 is correct.
 Flogging for Indian soldiers was abolished. So, statement 2 is correct.
 Tipu Sultan’s family were moved to Calcutta.
 It is believed that the brutal and swift suppressing of the Vellore Mutiny is partly
responsible for the Southern sepoys not taking part in the Indian Revolt of 1857.
So, option (d) is correct.

ADDITIONAL INFORMATION:

 The Vellore mutiny on 10 July 1806 was the first instance of a large-scale and violent mutiny
by Indian sepoys against the East India Company, predating the Indian Rebellion of 1857 by
half a century.
 The revolt, which took place in the South Indian city of Vellore, lasted one full day, during
which mutineers seized the Vellore Fort and killed or wounded 200 British troops.
 The mutiny was subdued by cavalry and artillery from Arcot. Total deaths amongst the
mutineers were approximately 350; with summary executions of about 100 during the
suppression of the outbreak, followed by the formal court-martial of smaller numbers

23
A SERIES

The major causes for the Vellore mutiny are stated below:

 The English disregard to the religious sensitivities of the Hindu and Muslim Indian sepoys.
 Sir John Craddock, the Commander-in-Chief of the Madras Army had issued orders prohibiting
soldiers from wearing religious marks on their foreheads and also to trim their moustaches
and shave off their beards. This offended both Hindu and Muslim soldiers.
 They were also asked to wear new round hats instead of the traditional headgear that they
were used to. This led to suspicion among the sepoys that they were being converted to
Christianity.
 Craddock was acting against warning from the military board not to bring about changes in
the military uniform without taking into consideration all required precautions of Indian
sensibilities.
 A few sepoys who had protested against these new orders were taken to Fort St. George and
punished severely. They were given heavy flogging.
 Also present in the Vellore Fort were the wife and children of Tipu Sultan (who was killed in
the Battle of Seringapatam in 1799) who were housed in a palace within the fort. Tipu Sultan’s
sons also instigated the rebellion.

20. Consider the following statements with reference to Doctrine of Lapse:


1. Those states which, directly or indirectly, were created by the English would not be allowed to
adopt heirs to their throne
2. Independent states were free to adopt their heirs.
3. The doctrine of lapse was applied to annex Lower Burma after the annexation of Upper Burma.
Which of the statements given above is/are correct?
(a) 3 only
(b) 1 and 2 only
(c) 2 and 3 only
(d) 1, 2 and 3

EXPLANATION:

In simple terms, the doctrine stated that the adopted son could be the heir to his foster
father’s private property, but not the state; it was for the paramount power (the British) to
decide whether to bestow the state on the adopted son or to annex it. Those states which,
directly or indirectly, were created by the English would not be allowed to adopt heirs to
their throne. So, statement 1 is correct.
Independent states which were not under the control of the British were free to adopt their
heirs. So, statement 2 is correct.
In 1852, commercial disputes in Rangoon prompted new hostilities between the British
and the Burmese. After the end of the second Burmese War (1852), Dalhousie annexed
Lower Burma with its capital at Pegu. But, the doctrine of lapse was not applied to the
annexation of Burma. So, statement 3 is not correct.

24
A SERIES

ADDITIONAL INFORMATION:

 In simple terms, the doctrine stated that the adopted son could be the heir to his foster father’s
private property, but not the state; it was for the paramount power (the British) to decide
whether to bestow the state on the adopted son or to annex it.
 The doctrine was stated to be based on Hindu law and Indian customs, but Hindu law seemed
to be somewhat inconclusive on this point, and the instances of an Indian sovereign annexing
the state of his vassal on account of ‘lapse’ (i.e., leaving no issue as heir) were rather rare.
 Maharaja Ranjit Singh had annexed a few of his feudatory principalities on account of ‘lapse’.
Likewise, the Company in 1820 acquired a few petty Cis-Sutlej states on the absenceof heirs.
 Nonetheless, there was no clear-cut instance of an adopted son being deprived of an entire
state or of such astate being regarded as a ‘lapse’.
 It was a matter of chance that during Lord Dalhousie’s term many rulers of states died without
a male issue and sevenstates were annexed under the Doctrine of Lapse.
 The most important of these were Satara (1848), Jhansi and Nagpur (1854). The other small
states included Jaitpur (Bundelkhand), Sambhalpur (Orissa), and Baghat (Madhya Pradesh).
 Lord Dalhousie annexed Awadh in 1856 after deposingNawab Wajid Ali Shah on grounds of
misgovernment.

21. Consider the following statements with reference to Deoband Movement:


1. It was established by Muhammad QasimNanautavi and Rashid Ahmad Gangohi.
2. It aimed at the welfare of Muslims through western education and support of the British
government
Which of the statements given above is/are correct?
(a) 1 only
(b) 2 only
(c) Both 1 and 2
(d) Neither 1 nor 2

EXPLANATION:

The Deoband Movement was begun at the DarulUloom (or Islamic academic centre),
Deoband, in Saharanpur district (United Provinces) in 1866 by Mohammad QasimNanotavi
(1832-80) and Rashid Ahmed Gangohi (1828-1905) to train religious leaders for the Muslim
community. So, statement 1 is correct.
In contrast to the Aligarh Movement, which aimed at the welfare of Muslims through
Western education and support of the British government, the aim of the Deoband
Movement was moral and religious regeneration of the Muslim community. The instruction
imparted at Deoband was in the original Islamic religion. So, statement 2 is not correct.

ADDITIONAL INFORMATION:

The Deoband School (DarulUloom)

 The Deoband Movement was organised by the orthodox section among the Muslim ulema as a
revivalist movement with the twin objectives of propagating pure teachings of the Quran and
Hadis among Muslims and keeping alive the spirit of jihad against the foreign rulers.

25
A SERIES

 The Deoband Movement was begun at the DarulUloom (or Islamic academic centre), Deoband,
in Saharanpur district (United Provinces) in 1866 by Mohammad QasimNanotavi (1832-80)
and Rashid Ahmed Gangohi (1828-1905) to train religious leaders for the Muslim community.
 In contrast to the Aligarh Movement, which aimed at the welfare of Muslims through Western
education and support of the British government, the aim of the Deoband Movement wasmoral
and religious regeneration of the Muslim community.
 The instruction imparted at Deoband was in original Islamic religion.
 On the political front, the Deoband school welcomed the formation of the Indian National
Congress and in 1888 issued a fatwa (religious decree) against Syed Ahmed Khan’s
organisations, the United Patriotic Association and theMohammaden Anglo-Oriental
Association.
 Some critics attribute Deoband’s support to the nationalists more to its determined opposition
to Syed Ahmed Khan than to anypositive political philosophy.

22. Which of the following were the factors which prompted the early Europeans to look for sea
routes to India and East Indies?
1. Overcome the monopoly of Arabians over land routes to trade with India.
2. Rise of Renaissance in Europe.
3. Zeal to spread Christianity.
4. Fall in demand for goods in Europe.
5. Internal feuds of succession in Indian princely states.
Select the correct answer using the code given below.

(a) 1,2 and 4 only


(b) 1, 2 and 3 only
(c) 1, 2, 4 and 5 only
(d) 3, 4 and 5 only
EXPLANATION:

European try to find out an alternative sea route to India Because it became necessary for
the Europeans to a direct sea route to the East because of the increase in demands of
cotton, silk and spices. Statement 1 is correct.

The Renaissance had generated a great spirit of adventure among the people of western
Europe. Statement 2 is correct.

The European powers arrived in India for commercial reasons, especially spices. But they
also started converting local Indians to Christianity. Statement 3 is correct.

Rise in demand for goods in Europe especially spices, gems prompted the early Europeans to
look for sea routes to India and east indies. Statement 4 is not correct.

Internal struggle within India was present from Ancient times so it not the reason for advent
of early Europeans in the 15th century. Statement 5 is not correct.

26
A SERIES

ADDITIONAL INFORMATION:

 The old trading route between the East and West came under Turkish control after the
Ottoman conquest of Asia and capture of Constantinople in 1453.

 In 1492 Colombus of Spain set out to reach India and discovered America instead.

 In 1498, Vasco D agama of Portugal discovered a new sea route from Europe to India.

 In the 16th century the trade was monopoly of Spain and Portugal. Later it was dominated by
Dutch, French, and British after 1650.

 In 1505 Francisco De Almeida became the First governor of Portugal. His policy was also
known as “Blue water policy”(cartaze system).

 The four important ports Mangalore, Cannanore, Cochin, Calicut were captured.

 The first Europeans come to India, were also the last to leave this land on 1961.

23. Which one of the following statements is not correct with reference to Lord Dalhousie?
(a) He was the youngest Governor-General of India
(b) He introduced the centralized control called “Non-Regulation System"
(c) He created a separate Public Works Department
(d) He laid foundation of the Calcutta Medical College
EXPLANATION:

Lord Dalhousie was the Youngest Governor-General of India (36 Years). So, option (a) is
not correct.
For the newly acquired territories, he introduced the centralized control called “Non-
Regulation system” under this system a commissioner was appointed for a newly acquired
territory. So, option (b) is not correct.
Prior to the period of Dalhousie military boards managed the Public Works Department.
Dalhousie established a separate department for this purpose. The Public Work:
Department was given the responsibility of constructing roads, Canals bridges etc. During
the period of Dalhousie, the Grand Trunk Road from Calcutta to Peshawar was
reconstructed. Ganges canal, the canals in Punjab and many bridges over different rivers
were constructed. So, option (c) is not correct.
Calcutta Medical College, officially Medical College and Hospital, Kolkata, is an Indian
medical school and hospital. The school was established in 1835 by Lord William Bentinck
as Medical College, Bengal during British Raj. So, option (d) is correct.

ADDITIONAL INFORMATION:

Lord Dalhousie 1848-1856

 Second Anglo-Sikh War (1848-49) and annexation of Punjab (1849).


 Annexation of Lower Burma or Pegu (1852).

27
A SERIES

 Introduction of the Doctrine of Lapse and annexation of Satara (1848), Jaitpur and
Sambhalpur (1849), Udaipur (1852), Jhansi (1853), Nagpur (1854) and Awadh (1856).
 “Wood’s (Charles Wood, President of the Board of Control) Educational Despatch” of 1854 and
opening of Anglo-vernacular schools and government colleges.
 Railway Minute of 1853; and laying down of first railway line connecting Bombay and Thane
in 1853.Telegraph (4000 miles of telegraph lines to connect Calcutta with Bombay, Madras
and Peshawar) and postal (Post Office Act, 1854) reforms.
 Ganges Canal declared open (1854); establishment of separate public works department in
every province.
 Widow Remarriage Act (1856).

24. Consider the following statements with respect to Dutch rule in India:
1. They established their first factory in Masulipatnam in 1605.
2. They were only interested in trade and not in empire building.
3. They were defeated by the French.
Which of the statements given above is/are correct?

(a) 1 and 2 only


(b) 1 only
(c) 2 and 3 only
(d) 1 and 3 only
EXPLANATION:

The Dutch founded their first factory in Masaulipatam in Andhra Pradesh in 1605.
Subsequently they also established trading Centres in various parts of India. Dutch Suratte
and Dutch Bengal were established in 1616 AD and 1627 AD respectively. Option 1 is
correct.

The main commercial interest of Dutch lay not in India but in the spice Islands of Indonesia.
They had no dream to establishing an Empire India. Therefore, the Dutch had lost their last
trading posts in India. So option 2 is correct.

They were defeated by English in the Battle of Chinsura or Biderra on 1759. Hence option 3
is not correct.

ADDITIONAL INFORMATION:

The Dutch East India Company was created in 1602 and permanent trading post was in Indonesia.
 In India, they established the first factory in Masulipattanam in 1605, followed by Pulicat in
1610, Surat in 1616, Bimilipatam in 1641 and Chinsura in 1653.

 With the Dutch arrival commodities need has been changed from spices to cotton. The most
important Indian commodities are traded were silk, cotton, indigo, rice, and opium.

 The Dutch brutally killed some English traders in Amboyna.

28
A SERIES

 This incident rivalry between the two Europeans countries after prolonged period both the
countries came to the compromise in 1667 by which British agreed to withdraw all claims on
Indonesia and the British retired from India to concentrate on their more profitable trade in
Indonesia.

25. With respect to Anglo-French rivalry, consider the following statements:


1. The first Carnatic war was famous for Treaty of Aix- La Chapelle.
2. The second Carnatic war exposed the internal rivalry among India rulers which became an
excuse for Europeans to interfere and conquer.
3. The third Carnatic war established the supremacy of British in India.
Which of the statements given above is/are correct?

(a) 1 only
(b) 1 and 2 only
(c) 2 and 3 only
(d) 1, 2 and 3
EXPLANATION:

In the Treaty of Aix-la-Chapelle (1748), Madras was given back to the British in exchange
for the French fortress of Louisbourg in North America, which the British had captured.
So Statement 1 is correct.

The Chanda Sahib and Muzaffar Jung, supported by the French, vying to become the Nawab
of Arcot. In 1751, Robert Clive led British troops to capture Arcot, and
successfully defend it. The war ended with the Treaty of Pondicherry and signed. Hence
Statement 2 is correct.

After the Seven Years' War in Europe in 1756, conflict between French and British forces in
India. the war was decided in the south, where the British Sir Eyre Coote successfully
defeated the French, at the Battle of Wandiwash in 1760. Statement 3 is also correct.

ADDITIONAL INFORMATION:

 British and French both had vision of establishing political power over the region.
 The political situation in india was uncertain. NizamAsaf Jah was engaged in the battle with
the Marathas. The south kingdom of Coromondel coast without any strong ruler. Therefore
conscious of the weaker position in India was favour an extension to build the power.
 The first Carnatic war ended in 1748 when the treaty of Aix-la-Chapelle was signed and bringing
the Austrian war of succession to a conclusion.
 It is fought in the battle of St. Thome on the bank of the river Adyar between the French and
English.

29
A SERIES

26. With reference to the Farman of 1717 granted by Mughal emperor Farruksiyar to the English
east India company, consider the following statements:
1. The British were allowed duty free trade in all the provinces of Mughal empire.
2. The Company was granted the right to issue passes (dastaks) for the movement of such duty
free goods in India.
3. The company could now use its own currency for trade throughout India.
Which of the statements given above is/are correct?

(a) 1 only
(b) 1 and 3 only
(c) 2 and 3 only
(d) 2 only
EXPLANATION:

FarrukhSiyar's Farman was a giving concessions to the English East Indian company and
allowing them tax-free monopoly trade freely, except for a yearly payment.

Statement 1 is not correct.

Dastak was a trade permit mostly issued to European traders by officials of the East India
Company. Hence Statement 2 is correct.

Farruksiyar allowed the East Indian company to use their own currency for trade.
Statement 3 is correct.

ADDITIONAL INFORMATION:

Farrukhsiyar was the Mughal emperor from 1713 to 1719 after he murdered Jahandar Shah the
aid of the Sayyid brothers. He followed a policy of religious tolerance by Abolishing Jaziya and
pilgrimage Tax.

 FarrukhSiyar's Farman of 1717 relating to the Subah of Bengal included the privileges listed
below:
 That all the goods and necessaries carried by the agents of the English company either by
land or water would be free of custom-duties on payment.
 If the goods of the company be stolen, every measure should be taken to recover the lost
goods and punish the thief.
 In their attempt to establish factories at any place, they should be provided with every
assistance.
 In the event of any native merchant or weaver becomes indebted to the company's factors
(agents), the amount should be paid back.
 Measures should be taken so that the boats owned or hired by them are not molested by
anyone.

30
A SERIES

27. Which of the following statements is notcorrect regarding the State of India’s Birds 2020
Report?
(a) It is India’s first of its kind report that highlights observations by birdwatchers.
(b) It was recently released at the 13th COP Convention on the Conservation of Migratory Species
of Wild Animals held at Gujarat.
(c) According to the report, India has witnessed a big decline in migratory shorebirds.
(d) The report also highlighted the decline in the population of Indian Peacock.

EXPLANATION:

It is India’s first of its kind report that highlights observations by birdwatchers form the
basis of the analyses. The project was started in May 2018 as a means to collect the actual
data of birds in India. The final report was released on February 17, 2020, at 13th COP to
the convention on migratory species held in Gujarat. So, option (a) is not correct.

The State of India’s Bird 2020 was released at the 13th Conference of Parties of the
Convention on the Conservation of Migratory Species of Wild Animals which is taking place
in Gandhinagar, Gujarat. So, option (b) is not correct.

According to the report, India has witnessed a big decline in migratory shorebirds, raptors,
Indian Vulture, Large-billed Lea Warbler, Curlew Sandpiper, Richard’s Pipit and While-
rumped Vulture. So, option (c) is not correct.

The State of India’s Birds 2020 Report said that numbers of Indian Peacock have increased.
The report also highlights that good growth has been observed in Indian Peacock’s
population. Indian Peacock has also mentioned as the species of ‘least concern’ on the
IUCN list. So, option (d) is correct.

ADDITIONAL INFORMATION:

Key Highlights of Report

 The report suggests that about 867 birds were assessed which made it clear that almost all the
species are declining.
 The report categorises 101 species as of High Conservation Concern, 59 based on their range
size and abundance trends, and an additional 42 based on their IUCN Red List status.
 According to the report, India has witnessed a big decline in migratory shorebirds, raptors,
Indian Vulture, Large-billed Lea Warbler, Curlew Sandpiper, Richard’s Pipit and While-rumped
Vulture.
 The report also highlights that some species have increased in numbers such as Glossy Ibis,
Rosy Starling, Ashy Prinia, and Feral Pigeon.
 Birds were divided into different categories – 101 birds as a high concern, 319 birds as
moderate concern and 442 birds as low concern species.
 The report said that the number of India’s domestic or house sparrow has decreased in large
cities but almost stable overall. The report said that there are certain reasons for decreasing
numbers of house sparrows such as lack of suitable nesting sites and lack of insect (a key part
of sparrow’s diet) population.
31
A SERIES

28. What is the primary objective of the mission Purvodaya, which was seen recently in News?
(a) It aims to develop Southern India as an automobile Hub
(b) It aims to develop Eastern India as an integrated steel hub.
(c) It aims to develop North Eastern India as a Gateway to the South East Asia.
(d) It aims to develop North Western India as a Solar Power Hub.

EXPLANATION:

Mission Purvodaya was launched in 2020 for the accelerated development of eastern India
through the establishment of an integrated steel hub in Kolkata, West Bengal. The focus
will be on eastern states of India (Odisha, Jharkhand, Chhattisgarh, West Bengal) and
northern part of Andhra Pradesh which collectively holds ~80% of the country’s iron ore,
~100% of coking coal and a significant portion of chromite, bauxite and dolomite reserves.
So, option (b) is correct.

ADDITIONAL INFORMATION:

Integrated Steel Hub

 The proposed Integrated Steel Hub, encompassing Odisha, Jharkhand, Chhattisgarh, West
Bengal and Northern Andhra Pradesh, would serve as a torchbearer for socio-economic growth
of Eastern India.
 The objective of this hub would be to enable swift capacity addition and improve overall
competitiveness of steel producers both in terms of cost and quality. In addition to increased
steel capacity, this hub would also help enhance best-in- class value addition capabilities.
 The Integrated Steel Hub would focus on 3 key elements:
o Capacity addition through easing the setup of greenfield steel plants
o Development of steel clusters near integrated steel plants as well as demand centres
o Transformation of logistics and utilities infrastructure which would change the socio-
economic landscape in the East
 These elements would be supported through additional enablers such as ensured availability
of raw materials, presence of supporting industries such as capital goods and well-established
avenues for skill development.

29. Consider the following regarding the Central Administrative Tribunal (CAT)
1. The CAT has its benched in all the states across the country.
2. The members of CAT have been drawn from administrative and judicial streams
3. A CAT tribunal has similar powers as are vested in a civil court.
Which of the statements given above is/are correct?

(a) 1 only
(b) 1 and 2 only
(c) 2 and 3 only
(d) 1, 2 and 3

32
A SERIES

EXPLANATION:

There are 17 Benches and 21 Circuit Benches in the Central Administrative Tribunal all
over India. So, statement 1 is not correct.

The CAT is a specialist body consisting of Administrative Members and Judicial Members
who by virtue of their specialized knowledge are better equipped to dispense speedy and
effective justice. A Chairman who has been a sitting or retired Judge of a High Court heads
the Central Administrative Tribunal. So, statement 2 is correct.

Under Section 17 of the Administrative Tribunal Act, 1985, the Tribunal has been conferred
with the power to exercise the same jurisdiction and authority in respect of contempt of
itself as a High Court. So, statement 3 is correct.

ADDITIONAL INFORMATION:

Central Administrative Tribunal

 The Central Administrative Tribunal had been established under Article 323 - A of the
Constitution for adjudication of disputes and complaints with respect to recruitment and
conditions of service of persons appointed to public services and posts in connection with the
affairs of the Union or other authorities under the control of the Government.
 In pursuance of Article 323-A, the Parliament has passed the Administrative Tribunals Act in
1985. The act authorises the Central government to establish one Central Administrative
Tribunal and the state administrative tribunals. This act opened a new chapter in the sphere
of providing speedy and inexpensive justice to the aggrieved public servants.

Functions

 It exercises jurisdiction only in relation to the service matters of the parties covered by the
Administrative Tribunals Act, 1985.
 The Tribunal is guided by the principles of natural justice in deciding cases and is not bound
by the procedure, prescribed by the Civil Procedure Code.
 Under Section 17 of the Administrative Tribunal Act, 1985, the Tribunal has been conferred
with the power to exercise the same jurisdiction and authority in respect of contempt of itself
as a High Court.

Independence of working

 The conditions of service of the Chairman and Members are the same as applicable to a Judge
of High Court as per the Administrative Tribunals (Amendment) Act, 2006.
 The orders of CAT are challenged by way of Writ Petition under Article 226/227 of the
Constitution before respective High Court in whose territorial jurisdiction the Bench of the
Tribunal is situated.

30. Which of the following statements is/are correct regarding the National Groundwater
Management Improvement Programme?
1. It aims to promote panchayat-led groundwater management.

33
A SERIES

2. It is a Central Sector Scheme of the Ministry of Jal Shakti to improve ground water
management through community participation
3. It is being implemented only in the peninsular river flowing states of India.
Select the correct answer using the code given below.

(a) 1 only
(b) 1 and 2 only
(c) 2 and 3 only
(d) 1, 2 and 3
EXPLANATION:

Atal Bhujal Yojana (ABHY) is also known as National Groundwater Management


Improvement Programme. The objective of Atal Bhujal Yojana is to promote panchayat-led
groundwater management. ABHY also aims to increase farmers’ incomes and improvement
in water use efficiency on a larger scale. It has another objective to improve cropping
patterns and use of groundwater at the community level. So, statement 1 is correct.

It is a Central Sector Scheme of the Ministry of Jal Shakti to improve ground water
management through community participation. The scheme has a total outlay of Rs.6000
crore and is to be implemented over a period of 5 years (2020-21 to 2024-25). So,
statement 2 is correct.

The scheme aims to improve ground water management through community participation
in identified priority areas in seven States, viz. Gujarat, Haryana, Karnataka, Madhya
Pradesh, Maharashtra, Rajasthan and Uttar Pradesh. Implementation of the scheme is
expected to benefit nearly 8350 Gram Panchayats in 78 districts in these States. So,
statement 3 is not correct.

ADDITIONAL INFORMATION:

 Atal Bhujal Yojana (ATAL JAL) is an initiative for ensuring long term sustainability of ground
water resources in the country.
 The Department of Water Resources, River Development & Ganga Rejuvenation, Ministry of Jal
Shakti is adopting a mix of 'top down' and 'bottom up' approaches in identified ground water
stressed blocks in seven states, representing a range of geomorphic, climatic and hydrogeologic
and cultural settings.
 ATAL JAL has been designed with the principal objective of strengthening the institutional
framework for participatory ground water management and bringing about behavioral changes
at the community level for sustainable ground water resource management.
 The scheme envisages undertaking this through various interventions, including awareness
programmes, capacity building, convergence of ongoing/new schemes and improved
agricultural practices etc.

34
A SERIES

ATAL JAL has two major components:

 Institutional Strengthening and Capacity Building Component for strengthening institutional


arrangements for sustainable ground water management in the States including improving
monitoring networks, capacity building, strengthening of Water User Associations, etc.
 Incentive Component for incentivising the States for achievements in improved groundwater
management practices namely, data dissemination, preparation of water security plans,
implementation of management interventions through convergence of ongoing schemes,
adopting demand side management practices etc.

31. Consider the following statements regarding the Eastern Zonal Council.
1. It is a statutory body established under the State Reorganisation Act, 1956.
2. It comprises the states of Bihar, Jharkhand, Odisha, and West Bengal.
3. Andaman and Nicobar Islands, Lakshadweep are the special invitees this Zonal Council.
Which of the statements given above is/are correct?

(a) 1 only
(b) 1 and 2 only
(c) 2 and 3 only
(d) 1, 2 and 3
EXPLANATION:

The Zonal Councils are the statutory (and not the constitutional) bodies. They are
established by an Act of the Parliament, that is, States Reorganisation Act of 1956. So,
statement 1 is correct.

Eastern Zonal Council is a zonal council that comprises the states of Bihar, Jharkhand,
Odisha, and West Bengal. The States have been grouped into six zones having an Advisory
Council to foster cooperation among these States. Five Zonal Councils were set up vide
Part-III of the States Reorganisation Act, 1956. So, statement 2 is correct.

Andaman and Nicobar Islands, Lakshadweep are not members of any of the Zonal
Councils. However, they are presently special invitees to the Southern Zonal Council. So,
statement 3 is not correct.

ADDITIONAL INFORMATION:

Zonal Councils

 The Zonal Councils are the statutory (and not the constitutional) bodies. They are established
by an Act of the Parliament, that is, States Reorganisation Act of 1956.
 The act divided the country into five zones- Northern, Central, Eastern, Western and Southern
and provided a zonal council for each zone.
 While forming these zones, several factors have been taken into account which include: the
natural divisions of the country, the river systems and means of communication, the cultural

35
A SERIES

and linguistic affinity and the requirements of economic development, security and law and
order.
 Each zonal council consists of the following members:
o Home Minister of Central government.
o Chief Ministers of all the States in the zone.
o Two other ministers from each state in the zone.
o Administrator of each union territory in the zone

 In addition to the above mentioned Zonal Councils, a North-Eastern Council was created by a
separate Act of Parliament, the North-Eastern Council Act of 1971. Its members include Assam,
Manipur, Mizoram, Arunachal Pradesh, Nagaland, Meghalaya, Tripura and Sikkim.
 These are advisory bodies that make recommendations with regard to any matter of common
interest in the field of economic and social planning between the Centre and States border
disputes, linguistic minorities, inter-State transport or matters connected with the
reorganisation of States.

32. In context of mid-18th century India, which of the following places did not comprise the
region Northern Circars?
(a) Mustafanagar

(b) Ellore

(c) Rajamundry

(d) Arcot

EXPLANATION:

Kondapalli was known with the name, Mustafanagar. It is


a town near Vijayawada in Krishna district of the Indian state of Andhra Pradesh.
Option (a) is correct.

Ellora are situated in state of Maharashtra, the city of Aurangabad. Option (b) is correct.

During British rule, the District of Rajahmundry was created in the Madras Presidency in
1823, also known as Rajamahendravaram, located at Andhra Pradesh. Option (c) is
correct.

Nawabs of Arcot who ruled the Carnatic region of South India between about 1690 and
1855. Option (d) is not correct.

ADDITIONAL INFORMATION:

Northern Circars was a division of British India's Madras Presidency. It consisted of a narrow slip
of territory lying along the western side of the Bay of Bengal in the present-day Indian states
of Andhra Pradesh and Odisha. Northern Circars
were Chicacole, Rajahmundry, Eluru, Kondapalli and Guntur.The Northern Circars were governed
as part of Madras Presidency till 1947, after which became India's Madras State.

36
A SERIES

33. Which of the following statements regarding the Central Consumer Protection Authority are
correct?
1. The objective of this authority is to promote, protect and enforce the rights of consumers as a
class.
2. It also acts as a special purpose court in India that deals with cases regarding consumer
disputes, conflicts and grievances.
3. It is established under the Consumer Protection Act, 2019.
Select the correct answer using the code given below.

(a) 1 and 2 only


(b) 1 and 3 only
(c) 2 and 3 only
(d) 1, 2 and 3
EXPLANATION:

The objective of the Central Consumer Protection Authority (CCPA) is to promote, protect
and enforce the rights of consumers as a class. It will be empowered to conduct
investigations into violation of consumer rights and institute complaints / prosecution,
order recall of unsafe goods and services, order discontinuation of unfair trade practices
and misleading advertisements, impose penalties on manufacturers/endorsers/publishers
of misleading advertisements. So, statement 1 is correct.

Consumer Court is a special purpose court in India that deals with cases regarding
consumer disputes, conflicts and grievances not CCPA. They are judiciary hearings set up
by the government to protect consumer rights. Its main function is to maintain the fair
practices & contracts by sellers. So, statement 2 is not correct.

The Central Consumer Protection Authority (CCPA) is established under the Consumer
Protection Act, 2019. The CCPA will have the right to impose a penalty on the violators and

37
A SERIES

passing orders to recall goods or withdraw services, discontinuation of the unfair trade
practices and reimbursement of the price paid by the consumers. The Central Consumer
Protection Authority will have an investigation wing to enquire and investigate such
violations. The CCPA will be headed by the Director-General. So, statement 3 is correct.

ADDITIONAL INFORMATION:

 Consumer Protection Act, 2019 is a law to protect the interests of the consumers. This act was
inevitable to resolve a large number of pending consumer complaints in consumer courts
across the country. It has ways and means to solve the consumer grievances speedily.
 The basic aim of the Consumer Protection Act, 2019 to save the rights of the consumers by
establishing authorities for timely and effective administration and settlement of consumers’
disputes.
 As per the act; a person is called a consumer who avails the services and buys any good for
self-use. Worth to mention that if a person buys any good and avail any service for resale or
commercial purpose, is not considered a consumer. This definition covers all types of
transactions i.e. online and offline.
 The act has the provision of the establishment of the Consumer Disputes Redressal
Commissions (CDRCs) at the national, state and district levels. The CDRCs will entertain
complaints related to;
o Overcharging or deceptive charging
o Unfair or restrictive trade practices
o Sale of hazardous goods and services which may be hazardous to life.
o Sale of defective goods or services
 The Central Consumer Protection Authority (CCPA) will have the power to impose fines on the
endorser or manufacturer up to 2-year imprisonment for misleading or false advertisement
 The act has defined the criteria of Consumer Disputes Redressal Commission (CDRCs). The
National CDRC will hear complaints worth more than Rs. 10 crores. The State CDRC will hear
complaints when the value is more than Rs 1 crore but less than Rs 10 crore. While the District
CDRC will entertain complaints when the value of goods or service is up to Rs 1 crore.

34. In the context of Mughal empire of 18th century, consider the following statements:
1. Ahmad shah Abdali defeated the Mughals in third battle of Panipat in 1761.
2. Bahadur Shah I was called the Shah-i- Bekhabar of Mughal empire.
3. Abdulla Khan and Hussain Ali were known as King makers of Mughal kingdom.
Which of the statements given above is/are correct?

(a) 1 and 2 only


(b) 2 and 3 only
(c) 2 only
(d) 1, 2 and 3

38
A SERIES

EXPLANATION:

Third Battle of Panipat fought between the Maratha Confederacy and the Durrani Empire on
January 14, 1761, resulted in a catastrophic defeat for the Marathas. . Statement 1 is not
correct.

He was popularly known as Shah Alam I and called Shahi-i- Bekhabar by Khafi Khan due to
his appeasement parties by grants of title and rewards. . Statement 2 is correct.

The two Sayyid brothers were Abdulla Khan and Hussain Ali Khan. The exertions of these
two brothers won FarrukhSiyar his empire. he two Sayyid brothers were Abdulla Khan and
Hussain Ali Khan. The exertions of these two brothers won FarrukhSiyar his empire. The
Sayyids at last in 1719 set up Muhammad Shah as the Emperor. . Statement 3 is correct.

ADDITIONAL INFORMATION:

 In 1747, The Durrani Empire was Established by Ahamed Shah Durrani the modern state of
Afghanisthan. He set up the capital Kandahar.
 He is incompetent ruler who left the state in the hands of Udham Bai.
 During this time, the Maratha Empire stretched from the Indus in the north to the southern
regions of India.

The sayyid Brothers Abdulla Shah and Hussain Ali is known as King Makers. Muhammed Shah
killed the Sayyid Brothers with the help of Nizam-ul-Mulk and he became the Wazir and founder of
Hyderabad State.

BAHADUR SHAH I:

The eldest Son of Aurangazeb became the Emperor and killed his brothers Muhammed Azam and
Kam Bakhsh.

 He had a good policy with others empire like Maratha, Rajputs, Jats.
 He released the Maratha Prince Shahu who the son of Shambaji.
 Khafi Khan gave the title of Shah-i-bekhabar to him.

35. Towards the end of 17th century and the first half of 18th century, Bengal prospered while
rest of India had to face several challenges. What were the reasons for this anomaly?
1. Geographical location of Bengal made it a commercial hub.
2. Capable and strong rule by Nawabs of Bengal.
3. Relative isolation to political turmoil in rest of the India.
Select the correct answer using the code given below.
39
A SERIES

(a) 1 only
(b) 1 and 2 only
(c) 2 and 3 only
(d) 1, 2 and 3
EXPLANATION:

Bengal, located along the numerous rivers in the delta, had production centers that could
easily transport goods to distant places giving it a commercial edge. So statement 1 is
correct.

The nawabs of Bengal, the strongest of decentralized administrators of Mughals, were capable
of establishing strong rule over this region. So statement 2 is correct.

West and north west of India were tumultuous due to foreign invasions and expansive activities
by Marathas. Bengal kingdom in the east of India was passively and politically stable than the
rest of India. So statement 3 is correct.

ADDITIONAL INFORMATION:

Bengal was the most fertile and richest of India’s provinces. Its industries and commerce were well
developed. The British East India Company had abundant interest in making Bengal their entry
point of administrative expanse as 60 per cent of the exports from Asia to Britain was from Bengal.

The political stability due to isolation from the turmoil in west of India due to foreign invasions and
formation of independent kingdom by nawab of Bengal due to weakening central authority of
Mughals made the kingdom a superior and inevitable realm of India.

By 1630’s British established factories in Dacca, Hooghly, Kasimbazar and Balasore due to influx of
various rivers like Ganga, Brahmaputra and Meghna along with their tributaries making the Ganges
delta the best locality. By 1690’s, with the foundation of Calcutta, they completed the commercial
settlement process in Bengal.

36. Which of the following were the factors responsible for Battle of plassey?
1. Rampant misuse of trade privileges by the company.
2. Fortification of calcutta without Nawab permission.
3. Abolition of duties on internal duty by Nawab
Select the correct answer using the code given below.

(a) 1 and 2 only


(b) 1 only
(c) 2 and 3 only
(d) 1 and 3 only

40
A SERIES

EXPLANATION:

The company officials began to use Dastak (pass for internal custom duty evasion) for personal
and illegal uses which stripped the treasures out of Bengal kingdom.So statement 1 is
correct.

Siraj-ud-Daulah sent letter to demolish forts of British East India Company and French East
India Company in Calcutta and Chandernagar respectively. Only British refused to demolish,
which initiated the Battle of Plassey. So statement 2 is correct

British East India Company officials didn’t oblige to the internal duties of the Nawab of Bengal
as the company thought Siraj would drastically reduce its trade privileges in Bengal in
collusion with French. So statement 3 is not correct.

ADDITIONAL INFORMATION:

Battle of Plassey(1757)

CAUSES AND COURSE OF THE BATTLE:

 British East India Company refused to demolish its fort in Calcutta even on receiving a
warning letter from Sirajuddaulah. The French East India Company accepted Siraj’s letter
and demolished its Chandernagar fort.
 Non- responsiveness and over exploitation of trade privileges by British made Siraj to house
and arrest some British officials in his fort which is known as ‘BLACK HOLE TRAGEDY’.
 On hearing this incident Robert Clive, from Madras, assumed command of an expeditionary
land force along with Admiral Watson’s naval force to attack the Nawab of Bengal.
 Before the battle, the company bribed Siraj’s officers and military officials; making the
Plassey battle nothing but a form of betrayal.

RESULT OF THE WAR:

 The battle of plassey ended with treaty of Alinagar between Nawab of Bengal and Robert
Clive, agreeing to restore Company’s factories, allowing fortification of Calcutta and restoring
former trade privileges.
 Rise of British East India Company’s superiority in India.
 Drain of wealth of Indian economy started from here and continued till 1947.
 Company favoring person; Mir Jaafar; was made to seat as ruler of Bengal.

37. With reference to Anglo- Mysore wars, consider the following pairs:
War Treaty
1. First Anglo - Treaty of
Mysore war Madras

41
A SERIES

2. Second - Treaty of
Anglo Seringapatnam
Mysore war
3. Third Anglo - Treaty of
Mysore war Mangalore
Which of the above given pairs is/are correctly matched?

(a) 1 only
(b) 1 and 2 only
(c) 1 and 3 only
(d) 3 only
EXPLANATION:

First Anglo Mysore war conceded with Treaty of Madras; a friendship treaty; between Haider Ali
and Lord Verelst. So statement 1 is correct.

Second Anglo Mysore war ended with Treaty of Mangalore signed between Tipu Sultan and
British East India Company on 1784. So statement 2 is not correct.

Third Anglo Mysore war ended with Treaty of Seringapatnam signed by Lord Cornwallis, Tipu
Sultan and the representatives of Nizam of Hyderabad and the Maratha Empire. So statement
3 is not correct

ADDITIONAL INFORMATION:

FIRST ANGLO MYSORE WAR (1767-1769):

 CAUSES – territorial ambition of both Haider and British.


 COURSE OF WAR- Marathas, Nizam and British allied and attacked Haider. Haider made
ally with Nizam and Marathas and attacked nawab of Arcot and then suddenly attacked close
to company’s territory Madras which was unexpected by British. Haider was pacified.
 RESULT- Treat of madras, a friendship treaty was signed.

SECOND ANGLO MYSORE WAR (1780-1784):

 CAUSES- Marathas attacked Mysore and Haider asked for Company help (because of
friendship treaty) to which the company refused. French gave a supporting arm which
kindled British to capture Mahe causing the war.
 COURSE OF WAR- Battle of Porto Novo (or ParangiPettai) between Haider Ali and Eyre Coot
in British side, resulted in Britsh being victorious. Haider died in cancer but his son Tipu
Sultan continued the war.
 RESULT – Neither victory nor defeat on both the sides. Therefore treaty of Mangalore, a
status quo treaty.

42
A SERIES

THIRD ANGLO MYSORE WAR (1790-1792):

 CAUSES- Rise of Tipu Sultan in the south and arrival of Cornwallis as governor-general of
Bengal (not India yet) lead to the natural rift.
 COURSE OF WAR- Tipu attacked Travancore state (ally of British). Cornwallis himself
commanded the British force (an unique act for governor general) with the support of Nizam
and Hyderabad.
 RESULT- Tipu defeated and surrendered leading to treaty of seringapatnam making him
renounce half of his territories and accept a huge war indemnity.
FOURTH ANGLO MYSORE WAR (1799):

 CAUSES- Rise of Tipu Sultan again with a good relationship with French (especially Napolean
Bonaparte) and arrival of Lord Wellesley as governor-general who asked Tipu to sign
subsidiary alliance but Tipu refused.
 COURSE OF WAR- Helped again by Nizams and Marathas, Commander Arthur Wellesley
(brother of governor general Richard Wellesley) who fought in the battle of waterloo; killed
Tipu in the battle field.
 RESULT- Major territory taken by the British leaving remaining territory to be restored to
Wodeyar dynasty. Hence 1799 Madras province was created(Mangalore to Madras except
Coorg)

38. With reference to the policy of ring fence adopted by British to annex Indian states, consider
the following statements:
1. The policy aimed at creating buffer states which were used to safeguard the company’s
territories.
2. It was adopted by Warren hastings and used in war against Marathas and Mysore.
3. The policy of Subsidiary alliance was an extended version of policy of ring fence.
Which of the statements given above is/are correct?

(a) 1 and 2 only


(b) 1 only
(c) 2 and 3 only
(d) 1, 2 and 3
EXPLANATION:

Policy of Ring Fence (1765-1813) was established to avoid threats from Afghans invaders and
Marathas by organising defence in the frontiers of Awadh.So statement 1 is correct.
Warren Hastings used this policy to protect Awadh region from external threats in war against
Marathas and Mysore, at the expense of the Nawab of Awadh. So statement 2 is correct
The allies in ring fence policy were required to maintain subsidiary forces to be organized and
maintained by the company but paid by the rulers, a prototype for subsidiary alliance. So
statement 3 is correct.

43
A SERIES

ADDITIONAL INFORMATION:

POLICY OF RING FENCE:

 Warren Hastings became governor general when the British had to encounter the powerful
combination of the Marathas, Mysore and Hyderabad.

 It was a policy of defending the neighbour’s frontiers to safeguard our own territories.

 The defence of Awadh constituted the defence of Bengal at that time. Hence, when British
were at war with Marathas, Mysore and Hyderabad they used the Nawab of Awadh’s
expenditure to provide a subsidiary troop of his own which was maintained by the British.

 The policy of Subsidiary alliance was an extended version of policy of ring fence which sought
to reduce indian princely states to a cradle of dependence in the hands of British.

39. Consider the following statements with respect to constitutional developments during early
British period:
1. The regulating act of 1773 made a beginning of parliamentary control over the government of
the company.
2. The Company’s territories in India were called ‘the British possession in India’ for the first time
in 1784.
3. The Charter act of 1853 ended the company’s monopoly over India’s trade.
Which of the statements given above is/are correct?

(a) 1 only
(b) 1 and 2 only
(c) 2 and 3 only
(d) 1,2 and 3
EXPLANATION:

Regulating Act of 1773, for the first time, gave the British cabinet the right to exercise control
over Indian affairs. So statement 1 is correct.

Pitt’s India Act of 1784 made Company became a subordinate department of the State. The
Company’s territories in India were termed ‘British possessions’. So statement 2 is correct.

The Charter act of 1813 ended Company’s monopoly over trade in India except the trade with
China and the trade in tea. So statement 3 is not correct.

44
A SERIES

ADDITIONAL INFORMATION:

The Regulating Act of 1773

 It recognized that the Company’s role in India extended beyond mere trade to administrative
and political fields, and introduced the element of centralized administration.
 For the first time, the British cabinet was given the right to exercise control over Indian
affairs.
 In Bengal, the administration was to be carried out by governor-general and a council
consisting of 4 members, representing civil and military government. They were required to
function according to the majority rule.
 A Supreme Court of judicature was to be established in Bengal with original and appellate
jurisdictions.
 The governor-general could exercise some powers over Bombay and Madras

Pitt’s India Act of 1784

 The Company became a subordinate department of the State. The Company’s territories in
India were termed ‘British possessions’.

 A Board of Control consisting of the chancellor of exchequer, a secretary of state and four
members of the Privy Council (to be appointed by the Crown) were to exercise control over the
Company’s civil, military and revenue affairs.

 In India, the governor-general was to have a council of three (including the commander-in-
chief), and the presidencies of Bombay and Madras were made subordinate to the governor-
general.

The Charter Act of 1813

 The Company’s monopoly over trade in India ended except the trade with China and the
trade in tea.

 The constitutional position of the British territories in India was defined explicitly for the
first time.

 Powers of the Board of Control were further enlarged.

 A sum of one lakh rupees was to be set aside for the revival, promotion and encouragement
of literature, learning and science among the natives of India, every year.

 The regulations made by the Councils of Madras, Bombay and Calcutta were now required
to be laid before the British Parliament.

 Christian missionaries were also permitted to come to India and preach their religion.
45
A SERIES

40. Consider the following statements regarding the Permanent settlement introduced by the
British East India Company:
1. It was introduced by Lord Cornwallis.

2. The Zamindar’s right of ownership was made hereditary and transferable.

3. The amount to be paid to the company varied depending on the production.

Which of the above statements is/are notcorrect?

(a) 1 only
(b) 2 and 3 only
(c) 1 and 3 only
(d) 3 only
EXPLANATION:

Lord Cornwallis under directions from the then British PM, William Pitt, proposed the
Permanent Settlement system in 1786. This came into effect in 1793, by the Permanent
Settlement Act of 1793. So statement 1 is correct.

British East India Company allowed Zamindars to transfer, sell or hereditarily pass on the
Zamindari rights. So statement 2 is correct.

Amount to be paid by the landlords was fixed. It was agreed that this would not increase in
future (permanent). So statement 3 is not correct.

ADDITIONAL INFORMATION:

PERMANENT SETTLEMENT/ZAMINDARI SYSTEM

 Lord Cornwallis and John Shore introduced the permanent settlement(also known as
zamindari system) in India along with James Grant in 1790 which was implemented in
1793.This was different from Warren hastings system of annual settlement and quinquile
system
 It was introduced first in the Bengal province and northern Circar region (between Kolleru
and Chilika lake).

NEED FOR ZAMINDARI SYSTEM

 No British officers in huge numbers at that time


 Language diversity
 Zamindars were prevalent from Mauryan times (but hereditary in nature). The British
appointed zamindars randomly and if already hereditary zamindars were present, they were
appointed and continued hereditarily (if no expulsion occurred).
46
A SERIES

REVENUE COLLECTION:

 Company appointed zamindars had to collect revenue from peasants which can be paid in
cash or commodity(zamindar would convert it to money to be paid to the company)
 Zamindar was given 11 per cent commission of the revenue, proprietorship and partial
judicial rights in the village.
 Land revenue to be collected by company was fixed (assessed scientifically) and remained
unchanged till 1947. Hence called permanent settlement system.
 Failing to pay the money

1) By the zamindar will change his position as zamindar and transfer the land to the
company on the eve of the same day (also called the sunset clause).
2) By the peasant will transfer the land to the zamindar.

 Zamindar exploited the peasants in good season as amount to be paid to him from the farmer
was not fixed and lead to illegal expulsion of peasant’s land rights

41. In the context of Modern India, consider the following statements with respect to advent of
Europeans to India:
1. Prince Henry of Portugal first European to reach Indian Ocean from Atlantic.
2. Vasco da Gama became the first European to reach India by Sea.
3. Portuguese were the first to establish their rule in India.
Which of the statements given above is/are correct?

(a) 1 and 2 only


(b) 1 and 3 only
(c) 2 and 3 only
(d) 3 only
EXPLANATION:

Portuguese explorer Vasco de Gama becomes the first European to reach India via the Atlantic
Ocean where he arrives at Calicut on the Malabar Coast. STATEMENT 1 IS NOT CORRECT.

Vasco Da Gama sailed from Lisbon, Portugal, in 1497 discovered the sea-route from
Europe to India via the Cape of Good Hope in Africa. STATEMENT 2 IS CORRECT.

Portuguese were the first to arrive in India for trade and started establish to rule followed by
Dutch, English, Danish and French. STATEMENT 3 IS CORRECT.

ADDITIONAL INFORMATION:
 India trade Relation with Europe go back to the Ancient days of the Greek and Asia was
carrying several routes.

47
A SERIES

 The old trading route between East and West came under Turkish control after Ottoman
conquest of Asia and capture the Constantinople in 1453.
 The first step were taken by Portugal and Spain were controlled by the Goverments begin to
search for geographical discoveries.
 Vasco da Gama arrived at Calicut an important seaport located in South-West India on 1498
AD by King Zamorin.
 In 1505, Francisco de Almeida was appointed as the first Portuguese governor in India his
policy being controlling the Indian Ocean was known as the “Blue Water Policy” followed by
Alfonso de Albuquerque who replaced Almeida as the governor in 1509 and captured Goa from
the Sultan of Bijapur in 1510.
 By the end of the 16th century, the Portuguese captured Goa, Daman, Diu, and Salsette and
the vast stretches along the Indian coast.
 In 1608 Captain William Hawkins reached Surat and arrived mughal court for set up an
factory, but mughal emperor Jahangir refused it.
 In the 17th century, they started discovery of Brazil Diverted colonizing activities of Portugal to
the west.
42. The Lucknow declaration, which was recently in news, is related with
(a) First India-Africa Defence Minister’s Conclave
(b) India USA defence Minister’s meeting
(c) Resolution to control anti-microbial resistance
(d) Convention on Conservation of Migratory Species.
EXPLANATION:

India and African nations adopted the ‘Lucknow Declaration’ during the first India-Africa
Defence Minister’s Conclave. Overall, 50 African nations signed the declaration along with
India at the conclave, which was organized for the very first time.

Lucknow Declaration states that all the signatories “commit to continue our collaboration
in the fields of peace and security including conflict prevention, resolution, management
and peace building through the exchange of expertise and training, strengthening regional
and continental early warning capacities and mechanisms, enhancing the role of women
in peacekeeping and propagating the culture of peace”. So, option (a) is correct.

ADDITIONAL INFORMATION:

Key highlights of the Lucknow declaration

 In Luck now Declaration, both sides committed themselves to fight against terrorism by not
allowing their land to be used by any such outfit. Both sided decided to take action in rooting
out terrorism in all its forms and manifestations, terrorist safe havens and infrastructure,
disrupting terrorist networks and eliminating financing channels and halting cross-border
movement of terrorists.
 Both the sides called for strengthening the UN Counter-Terrorism mechanisms and to ensure
strict compliance with the UN Security Council sanctions regime on terrorism and Urge the
48
A SERIES

international community to envisage the adoption of Comprehensive Convention on


International Terrorism in the UNGA.
 The Declaration also stated that all member countries “encourage enhanced cooperation
between India and Africa on the evolving concept of Indo-Pacific
 Both sides welcomed the AU (African Union) vision for peace and security in Africa that
coincides with India’s vision of SAGAR (Security and Growth for all in the Region) “Silence The
Guns- Creating conducive conditions for African Development” is the African Union’s theme of
the year. This vision is to achieve ‘a conflict-free Africa, prevent genocide, make peace a reality
for all and rid the continent of wars, violent conflicts, human rights violations, and
humanitarian disasters.
 Both the sides called for deeper cooperation in the defence industry through investments and
joint ventures, among other things. It also mentioned India’s contribution to defence and
security in the African continent.

43. Which of following cities of India have received the tag of ‘World Heritage City'?

1. Chennai
2. Jaipur
3. Hyderabad
4. Ahmedabad
Select the correct answer using the code given below.

(a) 1 and 2 only


(b) 2 and 4 only
(c) 2 and 3 only
(d) 2 only
EXPLANATION:

UNESCO World Heritage Site is a place that is recognized by the United Nations
Educational, Scientific and Cultural Organization as of distinctive cultural or physical
importance which is considered of outstanding value to humanity. UNESCO strives to
encourage the conservation, identification, and maintenance of natural and cultural
heritage around the world. This is exemplified the Convention regarding the Protection of
the World Cultural and Natural Heritage, accepted by UNESCO in 1972. There are 38 World
Heritage Sites located in India. These include 30 cultural sites, 7 natural sites, and 1
mixed-criteria site. Among which Ahmedabad and Jaipur cities received the tag of ‘World
Heritage City'. So, option (b) is correct.

ADDITIONAL INFORMATION:

UNESCO World Heritage Committee (WHC)

 The World Heritage Committee is composed of representatives of 21 States Parties to the World
Heritage Convention who meet annually.
 The Convention Concerning the Protection of the World Cultural and Natural Heritage is an
international agreement that was adopted by the General Conference of UNESCO in 1972.

49
A SERIES

 It is based on the premise that certain places on Earth are of outstanding universal value and
should therefore form part of the common heritage of humankind.
 It basically defines the kind of natural or cultural sites which can be considered for inscription
on the World Heritage List.
 The Committee is in charge of implementing the Convention.
 To date, 1,092 sites in 167 countries have been inscribed on the World Heritage List.

44. In the context of development of art and literature in 18th century India, ‘HeerRanjha’, a
romantic epic in Punjabi literature was composed by?
(a) Warris Shah
(b) Shah Abdul Latif
(c) Mirza Galib
(d) KanchanNambiar
EXPLANATION:

The Qissa of HeerRanjha by Warris Shah (1706–1798) is among the most tragic romances of
Punjabi qisse. So option (a) is correct.

Shah Abdul Latif is a Sindhi poet whose celebrated work is Risalo, a collection of poems. So
option (b) is not correct.

Mirza Ghalib is considered one of the greatest Urdu poets of all time. Mirza Ghalib basically a
poet also wrote prose, known as Qate Burhan and Charagh-e Dair. So option (c) is not correct.
KanchanNambiar introduced OotamThullal, folk dance of Kerala in the 18th century. So
option (d) is not correct.

45. Consider the following statements regarding the Vivad Se Vishwas scheme.
1. The Vivad Se Vishwas scheme offers to settle pending tax-related disputes in the context of
both direct and indirect taxes.
2. It offers a complete waiver on interest and penalty to the taxpayers who pay their pending taxes
within the stipulated time limit.
Which of the statements given above is/are correct?

(a) 1 only
(b) 2 only
(c) Both 1 and 2
(d) Neither 1 Nor 2
EXPLANATION:

The Vivad Se Vishwas scheme offers to settle pending tax-related disputes related to direct
taxes, not indirect taxes in a speedy manner. The Vivad se Vishwas Scheme is to do for
direct tax-related disputes exactly what SabkaVishwas did for indirect tax-related disputes.
So, statement 1 is not correct.

50
A SERIES

The scheme launched by the Finance Ministry offers a complete waiver on interest and
penalty to the taxpayers who pay their disputed taxes on or before March 31, 2020 (within
the stipulated time limit ). Its key objective is to collect taxes and at the same time reduce
litigation. So, statement 2 is correct.

ADDITIONAL INFORMATION:

Specifics of the Vivad Se Vishwas scheme

 A taxpayer would be required to pay only the amount of the disputed taxes and will get a
complete waiver of interest and penalty provided he pays by 31st March 2020.
 Those who avail this scheme after 31st March 2020 will have to pay some additional amount.
 However, the scheme will remain open only till June 30, 2020. The scheme also applies to all
case appeals that are pending at any level.

How much money is at stake?

 According to reports, over Rs 9 lakh crore worth of direct tax disputes are pending in the courts.
 The government hopes to recover a big chunk of this in a swift and simple way, while offering
the taxpayers the relief of not having to fight the case endlessly.
 For a government that is staring at a big shortfall in revenues, especially tax revenues, the
scheme makes a lot of sense.

46. What is the primary objective of Ek Bharat Shreshtha Bharat scheme?


(a) To facilitate the Non-Resident Indians to study in India
(b) To encourage the Indians to settle in the Developed Countries
(c) To bring back the stolen idols from the Indian Temples
(d) To celebrate the unity in diversity of our nation.
EXPLANATION:

The broad objectives of the Ek Bharat Shreshtha Bharat scheme are as follows:-

 To CELEBRATE the Unity in Diversity of our Nation and to maintain and strengthen
the fabric of traditionally existing emotional bonds between the people of our
Country; (so, option (d) is correct.)
 PROMOTE the spirit of national integration through a deep and structured
engagement between all Indian States and Union Territories through a year-long
planned engagement between States;
 To SHOWCASE the rich heritage and culture, customs and traditions of either State
for enabling people to understand and appreciate the diversity that is India, thus
fostering a sense of common identity;
 TO ESTABLISH long-term engagements and
 TO CREATE an environment which promotes learning between States by sharing
best practices and experiences.

51
A SERIES

ADDITIONAL INFORMATION:

Ek Bharat Shreshtha Bharat scheme

 “Ek Bharat Shreshtha Bharat” was announced by Hon’ble Prime Minister on 31st October,
2015 on the occasion of the 140th birth anniversary of Sardar Vallabhbhai Patel.
Subsequently, the Finance Minister announced the initiative in his Budget Speech for 2016-
17.
 Through this innovative measure, the knowledge of the culture, traditions and practices of
different States & UTs will lead to an enhanced understanding and bonding between the States,
thereby strengthening the unity and integrity of India.
 All States and UTs will be covered under the programme.
 There will be pairing of States/UTs at national level and these pairings will be in effect for one
year, or till the next round of pairings.
 The State/UT level pairings would be utilized for state level activities. District level pairings
would be independent of the State level pairings.
 The activity will be very useful to link various States and Districts in annual programmes that
will connect people through exchanges in areas of culture, tourism, language, education trade
etc. and citizens will be able to experience the cultural diversity of a much larger number of
States/UTs while realising that India is one.

47. Consider the following statements regarding the Gram Nyayalayas.


1. It can function as a mobile court at any place within the jurisdiction of such Gram Nyayalaya.
2. The Gram Nyayalayas will have only civil jurisdiction and not the criminal jurisdiction.
3. Gram Nyayalayas allow for conciliation of the dispute and settlement of the same in the first
instance.
Which of the statements given above are correct?

(a) 1 and 3 only


(b) 1 and 2 only
(c) 2 and 3 only
(d) 1, 2 and 3
EXPLANATION:

A Gram Nyayalaya have jurisdiction over an area specified by a notification by the State
Government in consultation with the respective High Court. The Court can function as a
mobile court at any place within the jurisdiction of such Gram Nyayalaya, after giving wide
publicity to that regards. So, statement 1 is correct.

The Gram Nyayalayas have both civil and criminal jurisdiction over the offences and nature
of suits specified in the First, Second and Third Schedule of the Act. So, statement 2 is
not correct.

Gram Nyayalayas allow for conciliation of the dispute and settlement of the same in the
first instance. In execution of a decree, the Court can allow special procedures following
rules of natural justice. So, statement 3 is correct.

52
A SERIES

ADDITIONAL INFORMATION:

Gram Nyayalayas

 Gram Nyayalayas were established for speedy and easy access to the justice system in the rural
areas across the country.
 In terms of Section 3(1) of the Act, it is for the State Governments to establish Gram Nyayalayas
in consultation with the respective High Courts.
 The Act authorizes the Gram Nyayalaya to hold mobile court outside its headquarters.

Features of the Gram Nyayalayas

 Gram Nyayalaya are established generally at headquarter of every Panchayat at intermediate


level or a group of contiguous panchayat in a district where there is no panchayat at
intermediate level.
 The Gram Nyayalayas are presided over by a Nyayadhikari, who will have the same power,
enjoy same salary and benefits of a Judicial Magistrate of First Class.
 Such Nyayadhikari are to be appointed by the State Government in consultation with the
respective High Court.

Jurisdiction

 A Gram Nyayalaya have jurisdiction over an area specified by a notification by the State
Government in consultation with the respective High Court.
 The Court can function as a mobile court at any place within the jurisdiction of such Gram
Nyayalaya, after giving wide publicity to that regards.
 The Gram Nyayalayas have both civil and criminal jurisdiction over the offences and nature of
suits specified in the First, Second and Third schedule of the Act.
 The pecuniary jurisdiction of the Nyayalayas are fixed by the respective High Courts.
 Appeals in criminal matter can be made to the Sessions Court in the respective jurisdiction
and in civil matters to the District Court within a period of one month from the date of
judgment.

Trials

 Gram Nyayalayas can follow special procedures in civil matters, in a manner it deem just and
reasonable in the interest of justice.
 Civil suits are proceeded on a day-to-day basis, with limited adjournments and are to be
disposed of within a period of six months from the date of institution of the suit.
 In execution of a decree, the Court can allow special procedures following rules of natural
justice.
 Gram Nyayalayas allow for conciliation of the dispute and settlement of the same in the first
instance.
 Gram Nyayalayas has been given power to accept certain evidences which would otherwise not
be acceptable under Indian Evidence Act.

48. Consider the following Pairs


Iconic sites Location of states

1. Sivasagar - Maharashtra
53
A SERIES

2. Adichanallur - Tamilnadu
3. Hastinapur - Haryana
4. Rakhigarhi - Haryana
Which of the pairs given above are correctly matched?

(a) 1, 2 and 3 only


(b) 1, 2 and 4 only
(c) 1, 3 and 4 only
(d) 2, 3 and 4 only
49. Consider the following statements with respect to Charter act of 1793 enacted by British
Parliament for Indian administration:
1. It laid the foundation of governance based on written laws in British India in place of personal
rule of past rulers.
2. The act created a new authority in England known as Board of control to oversee the company’s
administration.
3. The revenue administration was divorced from the judiciary functions and this led to
disappearing of the Maal Adalats.
Which of the statements given above is/are correct?

(a) 1 and 2 only


(b) 1 and 3 only
(c) 1 only
(d) 2 and 3 only
EXPLANATION:
This Act continued the company's rule over the British territories in India.The Governor-
General's power of over-ruling his council was affirmed, and extended over the Governors of
the subordinate presidencies. STATEMENT 1 IS CORRECT.
The composition of the Board of Control has been changed. It was have a President and two
junior members, who were not necessarily members of the Privy Council. STATEMENT 2 IS
NOT CORRECT.

His Act separated the revenue administration and the judiciary functions of the company
leading to the disappearance of Maal Adalats (revenue courts). STATEMENT 3 IS CORRECT.

ADDITIONAL INFORMATION:
The Charter Act 1793, was an Act of the Parliament of Britain which has issued:

 The Governor-General was granted extensive powers over the subordinate presidencies.
 Royal approval was mandated for the appointment of the Governor-General, the governors,
and the Commander-in-Chief.
 This Act continued the company's rule for another 20 years over the British territories in
India.
 It has been continued the company's trade monopoly except tea, opium with China.
54
A SERIES

 For the Indian literature and promotion of Science provided the financial grands by the
Governor-Generals.
 The act has given more power to the court in India under the British Subject.
 He was also given authority over the governors of Madras and Bombay.

50. With respect to economic impact of British rule in India, which of the following is not true?
1. It led to de industrialization along with loss of traditional livelihood for artisans and
handicrafts.
2. It led to increased urbanization because of transformation of village economy.
3. It led to commercialization of Indian agriculture.
4. It created a new class of absentee landlords.
Select the correct answer using the code given below.

(a) 1 and 2 only


(b) 3 only
(c) 2 only
(d) 2 and 4 only
EXPLANATION:

There was a sudden and quick collapse of the urban handicrafts, which had for centuries made
India’s name in the markets of the entire civilized world. So, there was a great fall in the
industrialization. STATEMENT 1 IS CORRECT.

Village economy had isolated and self-sufficient. The cotton and spinning industry were hitting
worst as the same Silk and Woollen textiles were no better fate to overcome. STATEMENT 2
IS NOT CORRECT.

British conquest led the de-Industrialisation of the country and increased the dependence of
the people on agriculture as per the report census. This increased the pressure on Agriculture
was one of the major poverty in India. STATEMENT 2 IS CORRECT.

An absentee landlordism in agriculture occurs when the owner of the land does not live
within the agricultural region of its land holding. It was one of the worst effects of
the zamindari system introduced by the British in 1793. STATEMENT 4 IS CORRECT.

ADDITIONAL INFORMATION:
The economic policies followed by the British led to the rapid transformation of India’s economy. They
always remained foreigner in the land, Exploiting Indian resources and carrying away wealth.
In the 18th century East Indian company and its servant on the craftsmen of Bengal to sell their goods
below the market price and hire their service below the prevailing wage.
The peasants were also impoverished under the British rule. The policy of Robert Clive and warren
hasting led to such a devasting.

55
A SERIES

The both temporary and permanent zamindari settlements were remained uneviable. By the end of
19th century, the money-lenders had become the major cause of growing poverty of the rural people.
The growing commercialization of agriculture also helped the money-Lenders to exploit the cultivators.
Hence the Indian Agriculture and Technology was stagnant due to the less responsible of the British
Indian rule.

51. Which one of the following statements regarding the Irrawaddy Dolphin isnotcorrect?
(a) Chilika is home to the only known population of Irrawaddy dolphins in India.
(b) It is a true river dolphin, not an oceanic dolphin, that lives only in rivers.
(c) It is classified as Endangered according to International Union for Conservation of Nature
(IUCN).
(d) These Dolphins are listed in Schedule I of Indian Wildlife (Protection) Act 1972.
EXPLANATION:

Chilika is home to the only known population of Irrawaddy dolphins in India and one of
only two lagoons in the world that are home to this species. So, option (a) is not correct.

The Irrawaddy dolphin is a euryhaline species of oceanic dolphin not river dolphin found
in discontinuous subpopulations near sea coasts and in estuaries and rivers in parts of
the Bay of Bengal and Southeast Asia. So, option (b) is correct.

It is classified as Endangered according to International Union for Conservation of Nature


(IUCN). The total population of these aquatic mammals in the world is estimated to be less
than 7,500. So, option (c) is not correct.

These Dolphins are listed in Schedule I of Indian Wildlife (Protection) Act 1972. 146
Irrawaddy dolphins were recently sighted in Chilika Lake of Odisha. So, option (d) is not
correct.

ADDITIONAL INFORMATION:

Chilika lake

 Chilika is Asia’s largest and world’s second largest lagoon.


 It spreads over Puri, Khurda and Ganjam districts of Odisha on the east coast of India, at the
mouth of the Daya River.
 It is separated from the Bay of Bengal by a small strip of sand.
 It is the largest wintering ground for migratory birds on the Indian subcontinent.
 In 1981, it was designated the first Indian wetland of international importance under the
Ramsar Convention.
 Satapada, a Sea Mouth Island where Chilika Lake meets the Bay of Bengal, is home to the
Irrawaddy dolphins.
 Nalbana Bird Sanctuary or Nalbana Island is the core area of the Chilika Lake.

56
A SERIES

52. Recently the Reserve Bank of India launched the 5-year financial inclusion (2019/2024).
Which of the following measures have been included into that strategy?
1. Every adult enrolled under the Pradhan Mantri Jan DhanYojna should be enrolled under an
insurance scheme and pension scheme
2. The Universal Basic Income Scheme must be implemented within 2022.
3. Every village shall have access to financial service provider within a radius of 5 km.
4. The Public Credit Registry shall be made completely operational by March 2022.
Select the correct answer using the code given below.

(a) 1, 2 and 3 only


(b) 1, 2 and 4 only
(c) 1, 3 and 4 only
(d) 2, 3 and 4 only
EXPLANATION:

The Reserve Bank of India (RBI), in consultation with other financial services regulatory
bodies, has launched a National Strategy for Financial Inclusion that will help provide
access to formal financial services in an affordable manner by 2024. Further, every willing
and eligible adult enrolled under the Pradhan Mantri Jan Dhan Yojana should be enrolled
under an insurance scheme and pension scheme. So, statement 1 is correct.

National Strategy for Financial Inclusion does not talk about the implementation of The
Universal Basic Income Scheme. So, statement 2 is not correct.

The strategy recommends increasing outreach of banking outlets of Scheduled Commercial


Banks, Payments Banks and Small Finance Banks to provide banking access to every
village within a five km radius or hamlet of 500 households in hilly areas. So, statement
3 is correct.

Public Credit Registry should be made fully operational by March 2022 so that authorised
financial entities can leverage on the same for assessing credit proposals from all citizens.
So, statement 4 is correct.

ADDITIONAL INFORMATION:

 The Reserve Bank of India (RBI), in consultation with other financial services regulatory bodies,
has launched a National Strategy for Financial Inclusion that will help provide access to formal
financial services in an affordable manner by 2024.
 Necessary infrastructure should be created to move towards a less cash society. Financial
service providers have to strengthen outreach through virtual modes including mobile apps
and move towards an increasingly digital and consent-based architecture for customer
onboarding.
 All the relevant details pertaining to the ongoing skill development and livelihood generation
programmes should be made available to the new entrants at the time of account opening.
 The details of the account holders, including unemployed youth and women who are willing to
undergo skill development might be shared to the concerned skill development centres or

57
A SERIES

livelihood mission. Awareness on financial literacy, managerial skills, credit and market
linkages should be developed by National Skill Development Mission.
 Financial literacy modules should be used for orientation of children, young adults, women,
new workers, entrepreneurs, senior citizens etc. Focus should be on process literacy along with
concept literacy and the reach of Centers for Financial Literacy should be expanded at every
block in the country.
 Internal Grievances Redressal Mechanism of financial service providers should be
strengthened and a robust customer grievance portal or mobile app should act as a common
interface for lodging, tracking and redressal status of the grievances.
 With advancements in Geo-Spatial Information Technology, a robust monitoring framework
leveraging on the said technology can be developed for monitoring progress under financial
inclusion with special emphasis given to aspirational districts, North Eastern Region and Left-
Wing Extremist affected Districts.
 A monitoring framework and a GIS dashboard also can be developed.

53. The anti-smog gun is used to clear which of the following pollutants from air?
1. Particulate Matter 2.5
2. Ozone
3. Sulphur dioxide
Select the correct answer using the code given below.

(a) 1 only
(b) 2 and 3 only
(c) 1 and 3 only
(d) 1, 2 and 3
EXPLANATION:

The anti-smog Gun is designed to create an ultra-fine fog consisting of very fine water droplets
(less than 10-micron size) of atomised water. These tiny water droplets are thrown in the larger
area with the help of high-speed fans, which absorb even the smallest dust particles in the air.
Consequently, the dust and pollution particles get clear from the environment and PM 10 and
2.5 also reduce. So, option (a) is correct.

ADDITIONAL INFORMATION:

 Anti-smog gun is a device designed to reduce air pollution by spraying atomised water into the
atmosphere so that all the dust and polluted particles gets clear from the environment Anti
Smog gun is designed to create a ultra-fine fog consisting of very fine water droplets( 10 Micron
size)
 These tiny water droplets are spread in larger area with the help of high speed fan and absorb
even the smallest dust particles in the air, yet fall to the ground without wetness
 The device, which is connected to a water tank and mounted on a vehicle, could be taken
across the city to spray water to settle dust and other suspended particles.
 The cost of the machine is Rs 13 lakh and it can cover an area measuring up to 27, 000 to 37,
000 square metres (sqm).

58
A SERIES

 The Supreme Court had ordered the installation of anti-smog guns at large construction sites
and other locations to reduce dust pollution.

54. Which of the following statements is/are correct regarding Western Ghats?
1. Western Ghats exhibit a higher degree of endemism than Eastern Ghats.
2. It is one of the hotspots of biological diversity in the world.
3. It is a UNESCO World Heritage Site.
Select the correct answer using the code given below.

(a) 1 and 2 only


(b) 2 and 3 only
(c) 1 and 3 only
(d) 1, 2 and 3
EXPLANATION:

A significant characteristic of the Western Ghats is the exceptionally high level of biological
diversity and endemism than eastern ghats. In particular, the level of endemicity for some of
the 4-5,000 plant species recorded in the Ghats is very high: of the nearly 650 tree species
found in the Western Ghats, 352 (54%) are endemic. Animal diversity is also exceptional, with
amphibians (up to 179 species, 65% endemic), reptiles (157 species, 62% endemic), and fishes
(219 species, 53% endemic). Invertebrate biodiversity, once better known, is likely also to be
very high (with some 80% of tiger beetles endemic). So, statement 1 is correct.

This mountain chain is recognized as one of the world’s eight ‘hottest hotspots’ of biological
diversity along with Sri Lanka. So, statement 2 is correct.

UNESCO's World Heritage Committee inscribed the Western Ghats of India as a world heritage
site. The sites are protected under the Convention on Protection of the World Cultural and
Natural Heritage with the assistance from the World Heritage Fund of UNESCO. So, statement
3 is correct.

ADDITIONAL INFORMATION:

 Western Ghats is a mountain range that runs parallel to the western coast of the Indian
peninsula, located entirely in India.
 It is a UNESCO World Heritage Site and is one of the eight "hottest hot-spots" of biological
diversity in the world.
 A total of 39 properties including national parks, wildlife sanctuaries were designated as world
heritage sites - twenty in Kerala, ten in Karnataka, five in Tamil Nadu and four in Maharashtra.
 It has over 7,402 species of flowering plants, 1,814 species of non-flowering plants, 139
mammal species, 508 bird species, 179 amphibian species, 6,000 insects species and 290
freshwater fish species.
 Thus, the demarcation of an ecologically sensitive area (ESA) is an effort to protect the fragile
eco-system from indiscriminate industrialisation, mining and unregulated development.
 Two committees – Gadgil and Kasturirangan – were appointed in the last eight years to identify
the areas that needed to be kept out from such activities.

59
A SERIES

55. The Central Vigilance Commissioner is selected on the recommendation of the Search
committee consisting
(a) The Prime Minister, the Speaker of the Lok Sabha and the Chief Justice of India.
(b) The President, the Chief Justice of India and the Prime Minister.
(c) The Prime Minister, the Union Home Minister and the Leader of Opposition in the Lok
Sabha.
(d) The Prime Minister, the Chief Justice of India and the leader of Opposition in the Lok Sabha.

EXPLANATION:

The Central Vigilance Commissioner and the Vigilance Commissioners shall be appointed
by the President on recommendation of a Committee consisting of the Prime Minister
(Chairperson), the Minister of home affairs (Member) and the Leader of the Opposition in
the House of the People. So, option (c) is correct.

ADDITIONAL INFORMATION:

 Central Vigilance Commission (CVC) is an apex Indian governmental body created in 1964 to
address governmental corruption. In 2003, the Parliament enacted a law conferring statutory
status on the CVC.
 It has the status of an autonomous body, free of control from any executive authority, charged
with monitoring all vigilance activity under the Central Government of India, advising various
authorities in central Government organizations in planning, executing, reviewing and
reforming their vigilance work.
 It was set up by the Government of India Resolution on 11 February 1964, on the
recommendations of the Committee on Prevention of Corruption, headed by Shri K. Santhanam
Committee, to advise and guide Central Government agencies in the field of vigilance.
 The Annual Report of the CVC not only gives the details of the work done by it but also brings
out the system failures which leads to corruption in various Departments/Organisations,
system improvements, various preventive measures and cases in which the Commission's
advises were ignored etc.
 The Commission shall consist of:
o A Central Vigilance Commissioner - Chairperson;
o Not more than two Vigilance Commissioners - Members.

56. Consider the following statements with respect to ‘Portfolio system’ introduced by the
British:
1. Under the system, member of the Viceroy’s council was made in-charge of one or more
departments of the government and was authorized to issue final orders on behalf of the council
on matters of his department.
2. It was introduced by Lord Hardinge.
Which of the statements given above is/are correct?

(a) 1 only
(b) 2 only

60
A SERIES

(c) Both 1 and 2


(d) Neither 1 nor 2
EXPLANATION:

Viceroy's Executive Council was expanded by the addition of not less than six and not more
than 12 additional members for the purposes of legislation. It also envisages that the member
in-charge of his department could issue final orders with regard to matters which concerned
his department. OPTION 1 IS CORRECT.

Lord Canning had introduced the Portfolio system and laid the foundation of Cabinet
government in India in 1859. OPTION 2 IS NOT CORRECT.

ADDITIONAL INFORMATION:
In this system, each member was assigned a portfolio of a particular department.
For legislative purposes, the Governor-General’s Council was enlarged. Now, there were to be between
6 and 12 additional members,
FUNCTIONS:

 Lord Canning nominated three Indians to the Council in 1862 namely, the Raja of Benares,
the Maharaja of Patiala and Sir Dinkar Rao.
 The Viceroy had the power to overrule the council if necessary. Final bill passing needed the
viceroy approval.
 The Governor-General also had the power to promulgate ordinances without the council’s
concurrence during emergencies.
 The Secretary of State for India in Britain could also dissolve any act passed by the Governor-
General’s Council.
 The legislative councils established by the act of 1861 possessed no real powers and many
weak rulers.
 Indians association non officials were the members of elite section only.

57. Consider the following statements with respect to development of education in British India:
1. The Charter Act of 1813 was the first step towards education being made an objective of the
British government.
2. A General committee on public instruction was set up to solve the Orientalist-Anglicist
controversy on education.
3. Lord Macaulay’s minute discarded the system of Vernacular education and favoured the
downward filtration theory.
Which of the statements given above is/are correct?

(a) 1 and 2 only


(b) 2 and 3 only

61
A SERIES

(c) 1 and 3 only


(d) 1, 2 and 3
EXPLANATION:
Indian reformers believed that to keep up with times, a modern educational system was needed
to spread rational thinking. Therefore the new Charter Act was created by British in 1813 it
was the first step towards education by the government. . STATEMENT 1 IS CORRECT.

The Anglicists argued that the government spending on education should be exclusively
modern studies in the general committee. STATEMENT 2 IS CORRECT.
Lord Macaulay’s minute believed in education a few upper and middle-class students. In the
course of time, education would trickle down to the masses. This was called the infiltration
theory. . STATEMENT 3 IS CORRECT.

ADDITIONAL INFORMATION:
Charles Grant and William Wilberforce, who were the activists, compelled the East India Company to
give new policy and make way for spreading education through English in order to teach western
literature and preach Christianity.
The East Indian company promotion of education were,
Fort William College by Wellesley, Sanskrit college by Jonathan Duncan, R.R. Roy established the
Calcutta College for the western education.
The company also set up three Sanskrit colleges in Delhi, Agra, Calcutta.
The Lord Macaulay’s Minute set in favor of Anglicists to teach the English as a medium of language.
The down filtration Theory was the interpreters between the government and masses, which is
neglected the elementary schools in mass education.

58. Consider the following statements regarding Lord William Bentinck:


1. He was the first Governor General of India.
2. He passed Bengal Sati regulation. 1829 to abolish the practice of sati from India.
3. He suppressed thugee from India.
4. He was in favour of Vernacular as the medium of instruction for education in India.
Which of the statements given above is/are correct?

(a) 1 and 2 only


(b) 2 and 4 only
(c) 1,2 and 3 only
(d) 1,3 and 4 only
EXPLANATION:

Lord William Bentinck served as Governor-General of India from 1828 to 1835. He has been
credited for significant social and educational reforms in India. STATEMENT 1 IS CORRECT.

62
A SERIES

William Bentinck consultation with the army and officials passed the Bengal Sati Regulation,
in 1829. STATEMENT 2 IS CORRECT.

He ended lawlessness by eliminating thugi who were organised gangs of


professional robbers and murderers. STATEMENT 3 IS CORRECT.

He introduced English as the language of instruction in India on the advice of his council
member, Thomas Babington Macaulay. STATEMENT 4 IS NOT CORRECT.

ADDITIONAL INFORMATION:
 Lord William Bentinck, governor-general of Bengal 1828 and of India 1833. He reformed the
finances, opened up judicial posts to Indians, and suppressed such practices as suttee, or
widow burning, and thugi or ritual murder by robber gangs.
 Bentinck also made English, instead of Persian, the language of the higher courts and
of higher education and arranged for financial aid to colleges, which were to be adapted to
the Western models.
 A treaty of perpetual friendship with Maharaja Ranjit Singh was made, Charter act 1833 was
passed by which East India Company ceased to be a trading company.
 The first reform done was to abolish the Provincial Courts of Appeal and Circuit Courts
altogether. This was done by a regulation passed in 1829.

59. Consider the following statements regarding the Fakir- Sanyasi Revolt:
1. The main cause of the revolt was the restrictions imposed by British on visits to the holy places.
2. Debi Chaudhurani’s was one of the main women participants in the revolt against British.
3. There was no Hindu- Muslim unity in the revolt.
4. Anandamath written by Bankim Chandra Chatterjee is based on Sanyasi revolt.
Which of the statements given above is/are correct?

(a) 1, 2 and 3 only


(b) 1, 2 and 4 only
(c) 2 and 3 only
(d) 1, 3 and 4 only

60. Consider the following statements with respect to PagalPanthis movement:


1. Pagal - panthis were a semi religious sect founded by Karam shah.
2. The movement was against the oppressive acts of Zamindars against the native tenants.
Which of the statements given above is/are correct?

(a) 1 only
(b) 2 only
63
A SERIES

(c) Both 1 and 2


(d) Neither 1 nor 2
EXPLANTION:
Karam Shah, a local Sufi fakir or mendicant, lead Pagal Panthis movement which followed
a syncretic mixed religion of Hinduism, Sufism and animism. So statement 1 is correct.
Mymensingh district of Bengal region where this movement took place was burdened by
heavy oppressive taxes and intensive restrictions by the British and furthermore exploitation
by the local zamindars . So statement 2 is correct.

ADDITIONAL INFORMATION:
Pagal Panthis was a socio religious movement against the oppressive acts of british and the local zamindars
in the tribal areas surrounding the regions of modern day Meghalaya and Bangladesh. The movement was
started by a local Sufi fakir or a mendicant named Karam shah who led the Garo and Hajong communities
to a peasant uprising .Karam Shah himself was inspired by his guru Majnu, who lead a similar uprising
with Hindu sanyasis and Sufi fakirs in Kanpur in 1760’s.

The movement generally took a religious overtone where monotheism, unity and non violence where the
spokes of guidance used by Karam shah. This provoked a quick resistance from orthodox religious leaders
who dubbed the new religion as Pagal Panthis or followers of mad sect.

The movement survived the death of Karam shah and became even fiercer under Tipu Shah (his son) who
went on to establish an independent administration for two years in Sherpur of Mymensingh district of
Bengal. He was finally arrested by British and jailed. The tax concession by British faded the memory of this
historic occurrence.

61. Consider the following statement regarding the civil rebellions and tribal uprisings of 18th
and 19th century:
1. They were massive in totality but wholly local in their spread.
2. Their aim was to establish popular form of government led by people’s representatives.
3. These rebellions established strong and valuable local traditions of resistance to British rule.
Which of the statements given above is/are correct?

(a) 1 only
(b) 1 and 3 only
(c) 2 and 3 only
(d) 1,2 and 3
EXPLANATION:

Civil rebellions and tribal uprisings of 18th and 19th century drew large number of
participants but, in fact, were localized and occurred at different times in different regions.
So statement 1 is correct.

64
A SERIES

Common cause was resentment against imposition of laws by the ‘foreign government’ or an
alien rule and wasn’t initiated to form a people’s government. So statement 2 is not correct.

These rebellions were centuries old in form and ideological-cultural content. The less
recalcitrant of these were pacified through concessions by the authorities. On the whole,
however, these rebellions were able to establish valuable traditions of local resistance to
authoritarianism So statement 3 is correct.

ADDITIONAL INFORMATION:
Characteristics of civil rebellions and tribal uprisings
1. These uprisings general represented the common cause and conditions in most cases
though separated in time and place.
2. Semi-feudal leaders wanted to restore earlier forms of rule and social relations as these
leaders were backward looking and traditional in outlook.
3. The rebellions and uprisings were a result of local causes and grievances and were also
localized in their consequences.
4. Most common cause was resentment against imposition of laws by the foreign
government that seemed to destroy the traditional socio economic framework.
5. Violence was seen against traders and money lenders who were seen as a extension of
colonial government.
6. May uprising were encouraged by messiah like figure who were seen as a frontier to
bring an end to the sufferings brought by outsiders.
7. These uprisings were on a back foot from the beginning in the sense of outdated arms
and equipments they possessed against the modern weapons used by their enemies.

62. Consider the following statements:


1. The Religious disabilities act of 1856 was one of the major causes of revolt of 1857.
2. The act made it compulsory for sepoys to serve anywhere outside India irrespective of their
religion or caste.
Which of the statements given above is/are correct?

(a) 1 only
(b) 2 only
(c) Both 1 and 2
(d) Neither 1 nor 2

EXPLANATION:

British had abandoned its policy of non-interference in the socio-religious life of the Indians
like religious Disabilities Act of 1850 which emphasized that the change in religion would not
debar a son from inheriting the property of his heathen father which was a major socio
religious factor of 1857 revolt. So statement 1 is correct.

65
A SERIES

General Service Enlistment Act of 1856, introduced by Charles Canning, required every
Indian soldier to go overseas for deployment. So statement 2 is not correct.

ADDITONAL INFORMATION:
 General service enlistment act of 1856 provided every Indian soldier in British Indian
army to serve overseas within or outside india when ordered.
 This was considered taboo for Hindu sepoys as they considered travelling overseas would
get them rid of their caste and religious identity.
 The act provided a jolt to other sepoys because it didn’t entitle additional benefits to
overseas travelling soldier too. This created storm in hearts of Indian soldiers which later
added fuel to the initiation of revolt of 1857.
 Religious Disabilities Act of 1850 declared that conversion of religion wouldn’t debar the
son from inheriting father’s property.
 This was seen as an attempt of Christian missionaries to convert Hindus.
 Religious Disabilities Act of 1850, Abolition of Sati 1829 and Hindu widows remarriage act
1856 were seen as British interference in Indian religion, custom and tradition

63. Which of the following were the reasons for the failure of revolt of 1857?
1. It was a revolt led by soldiers only and it did not have participation from other sections of
society.
2. There was a lack of Hindu- Muslim unity.
3. The revolt lacked effective leadership and organisation.
4. Lack of clear understanding of colonial rule.
Select the correct answer using the code given below.

(a) 1 and 3 only


(b) 2,3 and 4 only
(c) 1,3 and 4 only
(d) 3 and 4 only
EXPLANATION:

The complete picture of the sepoy mutiny is that it involved many section of the civilian
population and not just the sepoys. The discontent of the sepoys was just one cause of the
disturbance. So statement 1 is not correct.

There was a remarkable sense of unity and complete cooperation between Hindus and
Muslims at all levels- people, soldiers and leaders. All rebels acknowledged Bahadur Shah
Zafar, last Mughal ruler, as the emperor. So statement 2 is not correct.

The revolt lacked coordination and centralized leadership which was no match to British
opponent’s effective generalship with exceptional abilities. So statement 3 is correct.

66
A SERIES

The mutineers lacked a clear understanding of colonial rule because they lacked common
forward looking programme as they represented diverse elements with differing grievances
and lacked the concept of current politics of British East India company. So statement 4 is
correct.

ADDITIONAL INFORMATION:
CAUSES OF FAILURE OF REVOLT OF 1857:
 Not a pan india revolt.
 Lack of strong leader in the centre to guide the revolt.
 The soldiers used primitive weapons; on exhaustion of their cannons; when compared to their
opponents.
 No common cause and hence no sense of nationalism.
 Not all the soldiers participated (only Bengali and Awadhi soldiers participated).
 Not all classes of society participated. Some Indian rulers even supported the British and the
zamindars acted as ‘break waters’ of the revolt.

64. The Global Forest Resources Assessment is released by which of the following organizations?
(a) United Nations Environment Programme
(b) Food and Agriculture Organization
(c) Global Green Growth Institute
(d) International Union for Conservation of Nature
EXPLANATION:

The Global Forest Resources Assessment (FRA) reports on the status and trends of the world's
forest resources. It is led by the Forestry Department of the Food and Agriculture Organization
of the United Nations (FAO). The FRA reports the extent of the world’s forest area as well as
other variables, including land tenure and access rights, sustainable forest management
(SFM), legal and institutional frameworks for forest conservation, and sustainable use. So,
option (b) is correct.

ADDITIONAL INFORMATION:

Highlights of the Global Forest Resources Assessment 2020 report

 The rate of forest loss in 2015-2020 declined to an estimated 10 million hectares (mha), down
from 12 million hectares (mha) in 2010-2015, according to the FRA 2020.
 The FRA 2020 has examined the status of, and trends in, more than 60 forest-related variables
in 236 countries and territories in the period 1990–2020.
 The world lost 178 mha of forest since 1990, an area the size of Libya, according to the report.
 However, the rate of net forest loss decreased substantially during 1990–2020 due to a
reduction in deforestation in some countries, plus increases in the forest area in others through
afforestation.

67
A SERIES

 The largest proportion of the world’s forests were tropical (45 per cent), followed by boreal,
temperate and subtropical.
 The world’s total forest area was 4.06 billion hectares (bha), which was 31 per cent of the total
land area. This area was equivalent to 0.52 ha per person.
 Among the world’s regions, Africa had the largest annual rate of net forest loss in 2010–2020,
at 3.9 mha, followed by South America, at 2.6 mha.
 On the other hand, Asia had the highest net gain of forest area in 2010–2020, followed by
Oceania and Europe.
 However, both Europe and Asia recorded substantially lower rates of the net gain in 2010–
2020 than in 2000–2010.
 Oceania experienced net losses of forest area in the decades 1990–2000 and 2000–2010.
 More than 54 per cent of the world’s forests were in only five countries — the Russian
Federation, Brazil, Canada, the United States of America and China.
 The highest per cent of plantation forests were in South America while the lowest was in
Europe.

65. Which of the following species are declared to be extinct in the world?
1. Chinese Paddlefish
2. Asiatic Cheetah
3. Sumatran Rhino
Select the correct answer using the code given below.

(a) 1 only
(b) 2 only
(c) 3 only
(d) 1, 2 and 3
EXPLANATION:

The Chinese paddlefish (Psephurus gladius) also known as Chinese swordfish, is a reportedly
extinct species of fish that was native to the Yangtze and Yellow River basins in China. At
maturity, the species measured an average length of 3 m (9.8 ft), making it one of the largest
species of primarily freshwater fishes. So, option (a) is correct.

The Asiatic cheetah (Acinonyx jubatus venaticus), also known as Iranian cheetah, is a
Critically Endangered cheetah subspecies surviving today only in Iran. It once occurred from
the Arabian Peninsula and the Near East to the Caspian region, Kyzylkum Desert and India,
but has been extirpated there during the 20th century.

The Sumatran rhinoceros, also known as the hairy rhinoceros or Asian two-horned rhinoceros
(Dicerorhinus sumatrensis), is a rare member of the family Rhinocerotidae and one of five
extant species of rhinoceros. They are now critically endangered, with only five substantial
populations in the wild: four in Sumatra and one in Borneo.

68
A SERIES

ADDITIONAL INFORMATION:

International Union for Conservation of Nature (IUCN)

 IUCN is a membership union uniquely composed of both government and civil society
organisations.
 Created in 1948, it is the global authority on the status of the natural world and the measures
needed to safeguard it.
 It is headquartered in Switzerland.
 The IUCN Red List of Threatened Species, is the world's most comprehensive inventory of the
global conservation status of plant and animal species.
 It uses a set of quantitative criteria to evaluate the extinction risk of species. These criteria are
relevant to most species and all regions of the world.
 The IUCN Red List Categories define the extinction risk of species assessed. Nine categories
extend from NE (Not Evaluated) to EX (Extinct). Critically Endangered (CR), Endangered (EN)
and Vulnerable (VU) species are considered to be threatened with extinction.
 It is recognized as the most authoritative guide to the status of biological diversity.
 It is also a key indicator for the SDGs and Aichi Targets.
 When discussing the IUCN Red List, the official term “threatened” is a grouping of three
categories: Critically Endangered, Endangered, and Vulnerable.

 The pink pages include the critically endangered species.


 Green pages are used for those species that were formerly endangered but have now recovered
to a point where they are no longer threatened.

66. Consider the following statements regarding the Coastal Regulation Zone (CRZ) rules:
1. While the CRZ Rules are made by the Union Ministry of Environment and Forests,
implementation is to be ensured by the state governments.
2. In all Rules, the regulation zone has been defined as the area up to 500 m from the high-tide
line (HTL).
Which of the statements given above is/are correct?

(a) 1 only
(b) 2 only
(c) Both 1 and 2
(d) Neither 1 nor 2
EXPLANATION:

The coastal regulation zones have been declared by the Ministry of Environment, Forest and
Climate change under the Environment Protection Act 1986. While the CRZ Rules are made
69
A SERIES

by the Union environment ministry, implementation is to be ensured by state governments


through their Coastal Zone Management Authorities. So, statement 1 is correct.

As per the notification issued by the Ministry of Environment, the coastal land up to 500
meters from the High Tide Line (HTL) and up to 100 meters along banks of creeks, estuaries,
backwater and rivers subject to tidal fluctuations, has been defined as falling under the
Coastal Regulation Zone (CRZ). So, statement 2 is correct.

ADDITIONAL INFORMATION:

 Under the section 3 of Environment Protection Act, 1986 of India, Coastal Regulation Zone
notification was issued in February 1991 for the first time, for regulation of activities in the
coastal area by the Ministry of Environment and Forests (MoEF).
 As per the notification, the coastal land up to 500m from the High Tide Line (HTL) and a stage
of 100m along banks of creeks, lagoons, estuaries, backwater and rivers subject to tidal
fluctuations, is called the Coastal Regulation Zone (CRZ).
 CRZ along the country has been placed in four categories. The above notification includes only
the inter-tidal zone and land part of the coastal area and does not include the ocean part.
 The notification imposed restriction on the setting up and expansion of industries or processing
plaits etc. in the said CRZ. Coastal Regulation Zones (CRZ) are notified by the Government of
India in 1991 for the first time.
 Under this coastal areas have been classified as CRZ-1, CRZ-2, CRZ-3, CRZ-4. And the same
were retained for CRZ in 2003 notifications as well.
o CRZ-1: These are ecologically sensitive areas these are essential in maintaining the
ecosystem of the coast. They lie between low and high tide line. Exploration of natural
gas and extraction of salt are permitted
o CRZ-2: These areas are urban areas located in the coastal areas. Under Coastal Zone
Regulation (CRZ) Notification 2018, the floor space index norms has been de-freezed.
o CRZ-3: Rural and urban localities which fall outside the 1 and 2. Only certain activities
related to agriculture even some public facilities are allowed in this zone
o CRZ-4: This lies in the aquatic area up to territorial limits. Fishing and allied activities
are permitted in this zone. No Solid waste should be let off in this zone. This zone has
been changed from 1991 notification, which covered coastal stretches in islands of
Andaman & Nicobar and Lakshadweep.

67. Which of the following statements about Green India Mission is not correct?
(a) It is one of the eight missions launched under the National Action Plan on Climate Change.
(b) The mission also includes revival of wetlands as part of its objectives.
(c) The mission was converged with schemes like MGNREGA, National Afforestation Program,
CAMPA, etc.
(d) It is implemented by the joint efforts of Ministry of Environment and Forests and
Ministry of Rural Development.

70
A SERIES

EXPLANATION:

The National Mission for a Green India is one of the eight Missions under the National Action
Plan on Climate Change (NAPCC). It is designed to take positive steps toward the vulnerability
assessment associated with depleting natural resources and their impacts on livelihoods of
the local people. So, option (a) is not correct.

Improvement in quality of forest cover and ecosystem services of forests /non-forests,


including moderately dense, open forests, degraded grassland and wetlands (5 m ha) is one of
the objectives of the mission. So, option (b) is not correct.

Activities under Green India Mission will be implemented under convergence with Mahatma
Gandhi National Rural Employment Guarantee Act (MGNREGA), Compensatory Afforestation
Fund Management and Planning Authority (CAMPA) and the National Afforestation Programme
(NAP). So, option (c) is not correct.

At the national level, the Mission is set up as an autonomous Society under the aegis of the
Ministry of Environment and Forest (MoEF) to facilitate smooth implementation of the Mission.
So, option (d) is correct.

ADDITIONAL INFORMATION:

GIM- Goals

The Mission goals are as listed below:

 To increase forest/tree cover to the extent of 5 million hectares (mha) and improve quality of
forest/tree cover on another 5 mha of forest/non-forest lands; Separate sub-targets exists for
different forest types and ecosystems (eg. Wetland, grassland, dense forest etc.). For instance,
GIM aims at:
 Improvement in quality of forest cover and ecosystem services of forests /non-forests, including
moderately dense, open forests, degraded grassland and wetlands (5 m ha). The sub-targets
are:
o Moderately dense forest cover, but showing degradation: 1.5 m ha
o Eco-restoration of degraded open forests: 3 m ha
o Restoration of Grasslands: 0.4 m ha
o Restoration of Wetlands: 0.10 m ha
 Eco-restoration/afforestation of scrub, shifting cultivation areas, cold deserts, mangroves,
ravines and abandoned mining areas (1.8 m ha) with separate sub –targets for each one of
those.
 Improvement in forest and tree cover in urban/peri-urban lands (0.20 m ha)
 Improvement in forest and tree cover on marginal agricultural lands/fallows and other non-
forest lands under agro-forestry / social forestry (3 m ha)
 To improve/enhance eco-system services like carbon sequestration and storage (in forests and
other ecosystems), hydrological services and biodiversity; along with provisioning services like
fuel, fodder, and timber and non-timber forest produces (Minor forest produces or MFPs) etc
which are expected to result from the treatment of 10 m ha;

71
A SERIES

 To increase forest based livelihood income for about 3 million households in and around these
forest areas; and
 Enhanced annual CO2 sequestration by 50 to 60 million tonnes in the year 2020.

68. Consider the following statements:


1. No linguistic minority has been notified by the government till date.
2. The State can regulate the minority institutions in national interest.
Which of the statements given above is/are correct?

(a) 1 only
(b) 2 only
(c) Both 1 and 2
(d) Neither 1 nor 2
EXPLANATION:

The Constitution uses the word ’minorities’ in some articles but does not define it anywhere.
Article 29 has the word ‘minorities’ in its marginal heading but speaks of “any section of
citizens having a distinct language script and culture”. Article 30 speaks specifically of two
categories of minorities – religious and linguistic. But no linguistic minority has been notified
by the government till date. So, statement 1 is correct.

The national level minority communities are Muslims, Sikhs, Buddhists, Christians,
Zoroastrians (Parsis) and Jains. However, notifying minority communities within a state is the
subject matter of the concerned state government. Maharashtra government has declared
Jews as minority while West Bengal has not declared so. So, statement 2 is correct.

ADDITIONAL INFORMATION:

 The Constitution uses the word ’minorities’ in some articles but does not define it anywhere
 “The Constitution of India used the word ‘minorities’ or its plural form in some articles 29 to
30 and 350 A to 350 B.
 He said that Article 29 has the word ‘minorities’ in its marginal heading but speaks of “any
section of citizens having a distinct language script and culture”.
 This may be a whole community generally seen as a minority or group within a majority
community.
 Article 30 speaks specifically of two categories of minorities – religious and linguistic.
 The remaining two articles – 350 A and 350 B relate to linguistic minorities only.
 “As per clause (c) of section 2 of the National Commission for Minorities Act, 1992, five
communities are declared as minority communities viz Muslims, Christians, Sikhs, Buddhists
and Zoroastrians (Parsis)

72
A SERIES

69. The movement was a tribal uprising in the Chotanagar region of present day Jharkhand. It
was against the taxes imposed by Britishers and they staged a Satyagraha. They opposed
the zamindars, baniyas, the missionaries, the muslims and the British. It was a kind of
Sanskritisationmovement.Which of the following movements is being described here?
(a) Chuar uprising
(b) Munda revolt
(c) TanhaBhagat revolt
(d) Gond uprising
EXPLANATION:
Economic distress and famine made the Chuar tribe of Midnapore district of Bengal to take
up arms against the British from 1766-1772 and 1795-1816. So option (a) is not correct.
Munda rebellion or the Ulgulan revolt was started by Birsa Munda to end the Victorian rule
in a rampageous and vehement path and transform to Munda state which later became the
state of Jharkhand .So option (b) is not correct

The Tanha Bhagat revolt conflicted the taxes imposed on them in a civil obedience movement
fashion and started a Satyagraha path much before Gandhi. So option (c) is correct
Gond uprising was against the British government in Adilabad district of Telangana . So
option (d) is not correct

ADDITIONAL INFORMATION:

Tanha Bhagat Movement (1914-1919)

 Jatra Oraon lead the Tanha Bhagat movement for Oroans and Tanha bhagats sects in
Chotanagpur district of Bihar. They refuted the zamindars, the moneylenders, Christian
missionaries and British.

 They were active Gandhians (followers of Ahimsa) in their lifestyle and showed performed
satyagraha; much before the civil disobedience movement of Gandhi; against the oppressive
taxes on them by the British. They were even actively engaged with leaders like Mahatma
Gandhi and Babu Rajendra Prasad.
 They were actively involved in continuing their movement by stopping to plough their lands,
driving of evil spirits in the nights called as ghost drives and continuous gatherings to
enthusiastically continue the gathering of their sects.
 British eyed their movement with suspicion and announced their crowding or gatherings as
illegal.

70. The Eco Sensitive Zones are declared under which of the following laws or rules?
(a) Wildlife Protection Act of 1972
(b) Indian Forest Act of 1927
(c) Forest Rights Act, 2006
(d) Environment Protection Act, 1986

73
A SERIES

EXPLANATION:

The Environment (Protection) Act, 1986 does not mention the word "Eco-Sensitive Zones".
However, Section 3(2)(v) of the Act, says that Central Government can restrict areas in which
any industries, operations or processes or class of industries, operations or processes shall
not be carried out or shall be carried out subject to certain safeguards. Besides Rule 5(1) of
the Environment (Protection) Rules, 1986 states that central government can prohibit or
restrict the location of industries and carrying on certain operations or processes on the basis
of considerations like the biological diversity of an area, maximum allowable limits of
concentration of pollutants for an area, environmentally compatible land use, and proximity
to protected areas. The above two clauses have been effectively used by the government to
declare ESZs or EFAs. So, option (d) is correct.

ADDITIONAL INFORMATION:

 Eco-Sensitive Zones (ESZs) or Ecologically Fragile Areas (EFAs) are areas in India notified by
the Ministry of Environment, Forests and Climate Change (MoEFCC), Government of India
around Protected Areas , National Parks and Wildlife Sanctuaries.
 The purpose of declaring ESZs is to create some kind of "shock absorbers" to the protected
areas by regulating and managing the activities around such areas. They also act as a
transition zone from areas of high protection to areas involving lesser protection.

Activities Allowed in ESZs

 Prohibited activities: Commercial mining, saw mills, industries causing pollution (air, water,
soil, noise etc), establishment of major hydroelectric projects (HEP), commercial use of wood,
Tourism activities like hot-air balloons over the National Park, discharge of effluents or any
solid waste or production of hazardous substances.
 Regulated activities: Felling of trees, establishment of hotels and resorts, commercial use of
natural water, erection of electrical cables, drastic change of agriculture system, e.g. adoption
of heavy technology, pesticides etc, widening of roads.
 Permitted activities: Ongoing agricultural or horticultural practices, rainwater harvesting,
organic farming, use of renewable energy sources, adoption of green technology for all
activities.

71. Which of the following can be classified as technical textile?


1. Bulletproof jacket
2. Fishing nets
3. Carpets
4. Vehicle seat belts
Select the correct answer using the code given below.

(a) 1 only
(b) 2 and 4 only
(c) 1 and 3 only
(d) 1, 2, 3 and 4

74
A SERIES

EXPLANATION:

A technical textile is a textile product manufactured for non-aesthetic purposes, where


function is the primary criterion. Technical textiles include textiles for automotive applications,
medical textiles (e.g., implants), geotextiles (reinforcement of embankments), agrotextiles
(textiles for crop protection), and protective clothing (e.g., heat and radiation protection for fire
fighter clothing, molten metal protection for welders, stab protection and bulletproof vests,
and spacesuits). Fishing nets, Carpets and Vehicle seat belts are classified as technical textile
because of its various non-aesthetic purposes. So, option (d) is correct.

ADDITIONAL INFORMATION:

National Technical Textiles Mission

 The Mission would have a four year implementation period from FY 2020-21 to 2023-24.
 It will move into sunset phase after four years period.
 A Mission Directorate in the Min. of Textiles headed by an eminent expert in the related field
will be made operational.
 The Directorate will not have any permanent employment and there will be no creation of
building infrastructure for the Mission purpose.

Components of the mission

 Component-I: Promoting both (i) fundamental research at fibre level and (ii) application-based
research in geo-textiles, agro-textiles, medical textiles, mobile textiles and sports textiles and
development of bio¬degradable technical textiles.
 Component-II: Promotion and Market Development.
 Component-III: Export promotion of technical textiles and ensuring 10% average growth in
exports per year upto 2023-24. An Export Promotion Council for Technical Textiles will be set
up for this purpose.
 Component-IV: Promoting technical education at higher engineering and technology levels
related to technical textiles.

72. Consider the following statements:


1. He headed the Tattvabodhinisabha which was for the study of India’s past with a rational
outlook.
2. He was the founder in 1848 of the Brahmo religion, which today is synonymous
with Brahmoism.
3. He was also a member of Brahmosamaj.
4. He was against the Christian missionaries for their conversion activities.
Which of the following personalities is being referred to in the above given statements?

(a) Raja Ram Mohan Roy


(b) Debendranath Tagore

75
A SERIES

(c) Keshub Chandra Sen


(d) IshwarchandraVidyasagar
EXPLANATION:

Raja Rammohan Roy established the Brahmo Samaj in in 1928. The aim of the Brahmo Samaj
was to purify Hinduism and preach monotheism. So option (a) is not correct.

Debendranath Tagore was a Hindu philosopher and religious reformer, active in


the Brahmo Samaj and the founder in 1848 of the Brahmo religion. So option (b) is correct.

In 1865, this lean towards Christianity caused a rift between Keshub chandra sen and Tagore
and he broke away from the Samaj and formed the Bharat Barshiya Brahmo Samaj (Brahmo
Samaj of India). So option (c) is not correct.

shwar Chandra Vidyasagar worked tirelessly for social emancipation and as a social reformer,
he is remembered for his contribution towards the removal of social injustice, upliftment of
women, allowing widow remarriage and preached the ills of polygamy. So option (d) is not
correct.

ADDITIONAL INFORMATION:

DEBENDRANATH TAGORE:

 Joined the Brahmo samaj in 1842 and organized the Tattvaranjini Sabha at Calcutta to
encourage religious inquires and disseminate the essence of Upanishads.

 He confronted the immoderate attack of the Christian missionaries on the Indian culture,
customs and religion. He wrote ‘Brahmo Dharma’(inspired from Hindu scriptures) to counter
their radical enforcement.

 The growth of differences originated between old and the new groups of Debendranath Tagore
and Keshub Chandra Sen respectively which led to split in Brahmo Samaj in 1886.
Debendranath was conservative while Keshub was reformist and dynamic and thus the split
happened.

76
A SERIES

 The old group which was conservative in nature was known as Adi Brahmo Samaj. Therefore
Adi Brahmo Samaj was originally established by Raja Ram Mohan Roy but Debendranath
introduced it in practice.
Its doctrine was that “Brahmoism is the highest form of Hinduism”.
 The new group by KC Sen was known as Brahmo Samaj of India or Nava Vidhana which was
an all india movement in 1890’s. It believed that “Brahmoism is Catholic and Universal” and
also introduced Brahmopasna, a form of worship. Brahmo Samaj of India further split into
Sadharan Brahmo Samaj founded by Anandh Mohan Bose and Shivnath Shastri.

73. Consider the following statements regarding JyotibaPhule:


1. He founded the Society of Seekers of Truth to fight against caste discrimination.
2. His book ‘Gulamgin’ highlighted the historical roots of Shudras’ slavery and compared it with
slavery of the Blacks in America.
3. He is believed to be the first Hindu to start an orphanage for the unfortunate children.
Which of the statements given above is/are correct?

(a) 1 and 2 only


(b) 2 only
(c) 2 and 3 only
(d) 1,2 and 3
EXPLANATION:

Jyotiba Phule (1827-1890), born in Satara, Maharashtra, belonged to the mali (gardener)
community and organized a powerful movement against upper caste domination and
brahminical supremacy. Phule founded the Satyashodhak Samaj (Truth Seekers’ Society) in
1873, with the leadership of the samaj coming from the backward classes, malis, telis, kunbis,
saris and dhangars. So, statement 1 is correct.

He dedicated his book Gulamgiri or Gulamgin to the American movement to free slaves, he
linked the conditions of the black slaves in America with those of the lower castes in India.
This comparison contains an expression of hope that one day, like the end of slavery in
America, there would be an end to all sorts of caste discriminations in Indian society. So,
statement 2 is correct.

He opened an orphanage home to avoid infanticide. In this regard, he is believed to be the first
Hindu to start an orphanage for the unfortunate children. So, statement 3 is correct.

ADDITIONAL INFORMATION:

Jyotiba Phule (1827-1890)

 The main aims of the Satyashodhak Samaj (Truth Seekers’ Society) were (i) social service, and
(ii) spread of education among women and lower caste people.

77
A SERIES

 Phule’s works, Sarvajanik Satyadharma and Gulamgiri, became sources of inspiration for the
common masses. Phule used the symbol of Rajah Bali as opposed to the brahmins’ symbol of
Rama. Phule aimed at the complete abolition of the caste system and socio-economic
inequalities; he was against Sanskritic Hinduism.
 This movement gave a sense of identity to the depressed communities as a class against those
brahmins who used religion and the blind faith of the masses to exploit the masses for personal
monetary gain.
 Phule, a firm believer in gender equality, was a pioneer in women’s education; he with the help
of his wife, Savitribai, opened a girls’ school at Poona; he was a pioneer of widow remarriage
movement in Maharashtra and also opened a home for widows in 1854.
 Phule was awarded the title ‘Mahatma’ for his social reform work.

74. Consider the following statements in the context of 19th & 20th century socio-religious
movement:
1. The Muslim reform movements started later than the Hindu reform movements.
2. Islam was relatively freer from social evils and superstitions as compared to Hinduism.
3. Wahabi movement was in favour of introducing western education among Muslims.
Which of the statements given above is/are correct?

(a) 1 only
(b) 1 and 2 only
(c) 2 and 3 only
(d) 1 and 3 only

EXPLANATION:

The Muslim reform movements in India started after 1830’s such as Wahabi/Walliullah
Movement, Titu Mir‘s Movement, Faraizi Movement, Ahmadiyya Movement, Aligarh Movement.
The Hindu reform movements started earlier than Muslim reform movements in the early
1810’s such as Brahmo Samaj, Prarthana Samaj, Paramahansa Mandali, Arya Samaj. So,
statement 1 is correct.

Both Hinduism and Islam were dominated by superstitions. So, statement 2 is not correct.

Wahabi/Walliullah Movement was started by Shah Walliullah. He inspired this essentially


revivalist response to western influences and the degeneration which had set in among Indian
Muslims. So, statement 3 is not correct.

ADDITIONAL INFORMATION:

Wahabi/Walliullah Movement

78
A SERIES

 The teachings of Abdul Wahab of Arabia and the preachings of Shah Walliullah (1702-1763)
inspired this essentially revivalist response to Western influences and the degeneration which
had set in among Indian Muslims and called for a return to the true spirit of Islam.
 He was the first Indian Muslim leader of the 18th century to organise Muslims around the two-
fold ideals of this movement: (i) desirability of harmony among the four schools of Muslim
jurisprudence which had divided the Indian Muslims (he sought to integrate the best elements
of the four schools); (ii) recognition of the role of individual conscience in religion where
conflicting interpretations were derived from the Quran and the Hadis.
 The teachings of Walliullah were further popularised by Shah Abdul Aziz and Syed Ahmed
Barelvi who also gave them a political perspective. Un-Islamic practices that had crept into
Muslim society were sought to be eliminated.
 Syed Ahmed called for a return to the pure Islam and the kind of society that had existed in
the Arabia of the Prophet’s time. India was considered to be dar-ul-Harb (land of the kafirs)
and it needed to be converted to dar-ul-Islam (land of Islam).
 Initially, the movement was directed at the Sikhs in Punjab but after the British annexation of
Punjab (1849), the movement was directed against the British. During the
 1857 Revolt, the Wahabi’s played an important role in spreading anti-British feelings. The
Wahabi Movement fizzled out in the face of British military might in the 1870s.
75. Consider the following:
1. Indian Social conference
2. Poona Sarvajanik Sabha
3. Justice movement.
M.G Ranade is associated with which of the following?

(a) 1 and 2 only


(b) 2 and 3 only
(c) 1 and 3 only
(d) 1, 2 and 3
EXPLANATION:

Indian Social Conference Founded by M.G. Ranade and Raghunath Rao, the Indian Social
Conference met annually from its first session in Madras in 1887 at the same time and venue
as the Indian National Congress. It focussed attention on the social issues of importance; it
could be called the social reform cell of the Indian National Congress, in fact.

Indian Social Conference launched the ‘Pledge Movement’ to inspire people to take a pledge
against child marriage.

The Poona Sarvajanik Sabha was founded in 1867 by Mahadeo Govind Ranade and others,
with the object of serving as a bridge between the government and the people.

Justice Movement in Madras Presidency was started by C.N. Mudaliar, T.M. Nair and P.
Tyagaraja to secure jobs and representation for the non-brahmins in the legislature. In 1917,
the Madras Presidency Association was formed which demanded separate representation for
the lower castes in the legislature. So, option (a) is correct.

79
A SERIES

ADDITIONAL INFORMATION:

Justice party

 The root of the Dravidian movement lies in Brahmin-non-Brahmin conflict.


 The non-Brahmans started demanding their right and questioned the dominance of Brahmins
in each and every sphere.
 In 1916, Zamindars and Maharajas in order to counter growing clout of Brahmins in society
and politics, established South India Welfare Association in Madras Presidency.
 The organization had eminent non-Brahman leaders as Dr TM Nair, Theagaroya Chetty, Dr C
Natesa Mudaliar, Panagal’s Raja and Sir A Ramaswamy Mudaliar.
 It started publishing English newspaper ‘Justice’, with growing popularity the society came to
be known as Justice Party. This Justice party contested the election and for the first time in
1921 Indian cabinet was formed in Madras Presidency.
Self-Respect Movement

 This movement was started by E.V. Ramaswamy Naicker, a Balija Naidu, in the mid-1920s.
 The movement aimed at nothing short of a rejection of the brahminical religion and culture
which Naicker felt was the prime instrument of exploitation of the lower castes.
 He sought to undermine the position of brahmin priests by formalising weddings without
brahmin priests.

76. Consider the following statements:


1. Deva Samaj was founded by Shiv Narayan Agnihotri which emphaised the importance of guru
in promoting good behaviour.
2. Dharma Sabha founded by Radhakant deb supported the abolition of sati but was against
western education.
3. Bharat Dharma Mahamandala was an orthodox Hindu organization which was formed reaction
to teachings of Arya samaj and Ramkrishna mission.
Which of the statements given above is/are correct?

(a) 1 and 2 only


(b) 2 and 3 only
(c) 1 and 3 only
(d) 1,2 and 3

EXPLANATION:

Dev Samaj was founded in 1887 at Lahore by Shiv Narayan Agnihotri (1850- 1927), earlier a
Brahmo follower, Dev Sadan is a religious and social reform society. The society emphasised
on the eternity of the soul, the supremacy of the guru, and the need for good action. It called

80
A SERIES

for an ideal social behaviour such as not accepting bribes, avoiding intoxicants and non-
vegetarian food, and keeping away from violent actions. Its teachings were compiled in a book,
Deva Shastra. Agnihotri spoke against child marriage. So, statement 1 is correct.

Radhakant Deb founded Dharma Sabha sabha in 1830. An orthodox society, it stood for the
preservation of the status quo in socio-religious matters, opposing even the abolition of sati.
However, it favoured the promotion of Western education, even for girls. So, statement 2 is
not correct.

Bharat Dharma Mahamandala is an all-India organisation of the orthodox educated Hindus,


it stood for a defence of orthodox Hinduism against the teachings of the Arya Samajists, the
Theosophists, and the Ramakrishna Mission. So, statement 3 is correct.

ADDITIONAL INFORMATION:

Bharat Dharma Mahamandala

 An all-India organisation of the orthodox educated Hindus, it stood for a defence of orthodox
Hinduism against the teachings of the Arya Samajists, the Theosophists, and the Ramakrishna
Mission.
 Other organisations created to defend orthodox Hinduism were the Sanatana Dharma Sabha
(1895), the Dharma Maha Parishad in South India, and Dharma Mahamandali in Bengal.
 These organisations combined in 1902 to form the single organisation of Bharat Dharma
Mahamandala, with headquarters at Varanasi.
 This organisation sought to introduce proper management of Hindu religious institutions, open
Hindu educational institutions, etc.
 Pandit Madan Mohan Malaviya was a prominent figure in this movement.

77. Which of the following species have been declared as Endangered Migratory Species from
India at the recent 13th Convention on the Conservation of Migratory Species of Wild
Animals?
1. Asian Elephant
2. Great Indian Bustard
3. Jaguar
4. Bengal Florican
Select the correct answer using the code given below.
(a) 1, 2 and 3 only
(b) 1, 2 and 4 only
(c) 1, 3 and 4 only
(d) 2, 3 and 4 only

78. Which of the following were the objectives of Theosophical society?


81
A SERIES

1. To form a nucleus of the universal brotherhood of humanity without distinction of race, creed,
sex, caste, or colour.
2. To encourage the study of comparative religion, philosophy, and science.
3. To investigate the unexplained laws of nature and the powers latent in man.
Select the correct answer using the code given below.

(a) 1 only
(b) 2 only
(c) 1 and 3 only
(d) 1, 2 and 3
EXPLANATION:

The Theosophical society believed that a special relationship could be established between a
person’s soul and God by contemplation, prayer, revelation, etc. It accepted the Hindu beliefs
in reincarnation and karma, and drew inspiration from the philosophy of the Upanishads and
samkhya, yoga and Vedanta schools of thought.

It aimed to work for universal brotherhood of humanity without distinction of race, creed, sex,
caste or colour (so, statement 1 is correct). The society also sought to investigate the
unexplained laws of nature and the powers latent in man (so, statement 3 is correct). The
Theosophical Movement came to be allied with the Hindu renaissance. It opposed child
marriage and advocated the abolition of caste discrimination, uplift of outcastes, improvement
in the condition of widows. The beliefs of the Theosophical Society were a strange mixture of
religion, philosophy and occultism. so, statement 2 is correct

ADDITIONAL INFORMATION:

 The Theosophical Society was founded by Madame Blavatsky and Col. Olcott in New York in
1875. The founders arrived in India in January 1879, and established the headquarters of the
Society at Adyar near Madras.
 In 1888, Mrs. Annie Besant joined the Society in England. Her membership proved an asset of
greatest value to the Society.
 The Theosophical Society stood for making a comparative study of all oriental religions, but it
considered ancient Hinduism as the most profoundly spiritual religion in the world.
 Theosophy subscribed to the spiritual philosophy of Hinduism and its doctrine of Karma and
transmigration of the soul.
 The beliefs of the Theosophical Society were a strange mixture of religion, philosophy and
occultism.
 It preached universal brotherhood of men irrespective of distinctions of caste, creed, race or
sex. It promoted a spirit of national pride among the Indians.
 In an atmosphere surcharged with proud assertions of the white man’s racial superiority and
denunciations of Hinduism, renowned European Theosophists like Annie Besant proclaimed
India’s cultural supremacy and helped in arousing Indian nationalism.
 The Theosophical Society did commendable work in the field of education. Its most successful
venture in this direction was the opening of the Central Hindu College at Varanasi in 1898.
82
A SERIES

 The Society opened schools for boys, for women, for the depressed classes and also encouraged
participation in the Boy Scout movement.
 The Society opposed child marriage, advocated abolition of caste, the uplift of outcastes, and
the amelioration of the condition of widows.
 The Theosophical Society proved to be a very potent factor “in the awakening and self-respect
of Indians”.

79. Consider the following pairs:


Regional Founder
kingdom
1. Awadh - Saadat Khan Burhan-ul-
Mulk
2. Bengal - Alivardi Khan
3. Hyderabad - Nizam-ul-MulkAsafJha
Which of the pairs given above is/are correctly matched?

(a) 1 and 2 only


(b) 2 and 3 only
(c) 1 and 3 only
(d) 1, 2 and 3

80. Consider the following pairs:


Foreign company Settlement

1. French - Masulipatnam
2. Danish - Serampore
3. Dutch - Chinsura
Which of the pairs given above is/are correctly matched?
(a) 1 and 2 only
(b) 2 and 3 only
(c) 1 and 3 only
(d) 1, 2 and 3
EXPLANATION:

The French settlements in India were Pondichery, Masulipatnam, Karikal, Yanaon (Andhra
Pradesh) on the Coromandel Coast, Mahé on the Malabar Coast and Chandernagor in Bengal.
So, pair 1 is correct.

The Danish East India Company was established in 1616 and, in 1620, they founded a factory
at Tranquebar near Tanjore, on the eastern coast of India. Their principal settlement was at
Serampore near Calcutta. So, pair 2 is correct.

After their arrival in India, the Dutch founded their first factory in Masulipatnam (in Andhra)
in 1605. Their other principal factories in India were at Surat (1616), Bimlipatam (1641),

83
A SERIES

Karaikal (1645), Chinsura (1653), Baranagar, Kasimbazar (near Murshidabad), Balasore,


Patna, Nagapatam (1658) and Cochin (1663). So, pair 3 is correct.

ADDITIONAL INFORMATION:

 English East India Company was formed on December 31, 1600 by the charter issued by Queen
Elizabeth I, which gave the company monopoly to trade in the East Indies for 15 years.
Settlements in India

 With Captain Thomas Best’s victory over the Portuguese (1612), the English established their
first factory at Surat (1613). Subsequently Sir Thomas Roe secured permission from Jehangir
to establish factories at Agra, Ahmedabad and Broach.
 Bombay came under the control of the Company, with Charles II (who received it as a part of
the Portuguese dowry) leasing it out to the English Company for an annual rent of 10 pounds.
 Madras with the Fort St. George replaced Masulipatnam as the English headquarters on the
east coast, when the former was given by the Chandragiri chief to the English in 1639.
 The city of Calcutta grew from the development of three villages Sutanuti, Gobindapur and
Kalikata secured from the Mughal governor of Bengal. The fortified settlement was named Fort
William (1700) and it became the seat of British power in India till 1911.
81. Who among the following wrote under the pen-name of ‘Lokahitawadi’?
(a) BalShastriJambekar
(b) Vishnu Shastri Pundit
(c) RG Bhandarkar
(d) GopalHariDeshmukh
EXPLANATION:

Balshastri Jambhekar is also known as Father of Marathi journalism for his efforts in starting
journalism in Marathi language with the first newspaper in the language named 'Darpan' in
the early days of British Rule in India. So, option (a) is not correct.

Vishnushastri Pandit worked as a translator in British government. He became very active in


Widow Marriage [Vidhava Vivah] Movement. On 28 January 1866, Vishnushastri, inspired by
Phule’s movement opened an institution to promote widow remarriage. The institution (a
society) was known as Punar Vivahtojak Mandal(remarriage association). So, option (b) is not
correct.

Sir Ramakrishna Gopal Bhandarkar was an Indian scholar, orientalist, and social reformer.
In 1853, while a student, Bhandarkar became a member of the Paramhansa Sabha, an
association for furthering liberal ideas which was then secret to avoid the wrath of the powerful
and orthodox elements of contemporary society. So, option (c) is not correct.

Gopalhari Deshmukh (1823-1892) was a social reformer and rationalist from Maharashtra. He
held the post of a judge under British raj, but wrote for a weekly Prabhakar under the pen
name of Lokahitawadi on social reform issues. So, option (d) is correct.

84
A SERIES

ADDITIONAL INFORMATION:

Gopalhari Deshmukh ‘Lokahitawadi’

 Gopalhari Deshmukh (1823-1892) was a social reformer and rationalist from Maharashtra. He
held the post of a judge under British raj, but wrote for a weekly Prabhakar under the pen
name of Lokahitawadi on social reform issues.
 He advocated a reorganisation of Indian society on rational principles and modern, humanistic,
secular values.
 He attacked Hindu orthodoxy and supported social and religious equality.
 He wrote against the evils of the caste system. He said, “If religion does not sanction social
reform, then change religion.”
 He started a weekly, Hitechhu, and also played a leading role in founding the periodicals, Gyan
Prakash, Indu Prakash and Lokahitawadi.

82. Who among the following was the author of the work ‘Stripurushtulna’?
(a) Rashsundari Debi
(b) PanditaRamabai
(c) TarabaiShinde
(d) SavitribhaiPhule
EXPLANATION:

Rashsundari Devi (1800-1890) was born in West Bengal. At the age of 60, she wrote her
autobiography in Bangla. Her book titled ‘Amar Jiban’ is the first known autobiography written
by an Indian woman. So, option (a) is not correct.

Pandita Ramabai Sarasvati was a women's right & education activist, a pioneer in the
education and emancipation of women in India, and a social reformer. She was the first woman
to be awarded the titles of Pandita as a Sanskrit scholar and Sarasvati after being examined
by the faculty of the University of Calcutta. In the late 1890s, she founded Mukti Mission at
Kedgaon village, forty miles east of the city of Pune. The mission was later named Pandita
Ramabai Mukti Mission. So, option (b) is not correct.

Tarabai Shinde was a feminist activist who protested patriarchy and caste in 19th century
India. She is known for her published work, Stripurush Tulana ("A Comparison Between
Women and Men"). The pamphlet is a critique of upper-caste patriarchy, and is often
considered the first modern Indian feminist text. So, option (c) is correct.

Savitribai Phule was an Indian social reformer, educationalist, and poet from Maharashtra.
She is regarded as the first female teacher of India. Along with her husband, Jyotirao Phule,
she played an important and vital role in improving women's rights in India. She is regarded
as the mother of Indian feminism. So, option (d) is not correct.

ADDITIONAL INFORMATION:

85
A SERIES

 Tarabai Shinde (1850–1910) was a feminist activist who protested patriarchy and caste in 19th
century India.
 She is known for her published work, Stripurush Tulana ("A Comparison Between Women and
Men"), originally published in Marathi in 1882.
 The pamphlet is a critique of upper-caste patriarchy, and is often considered the first modern
Indian feminist text.
 It was very controversial for its time in challenging the Hindu religious scriptures themselves
as a source of women's oppression, a view that continues to be controversial and debated
today.
 Shinde was an associate of social activists Jotirao and Savitribai Phule and was a founding
member of their Satyashodhak Samaj ("Truth Finding Community") organisation.
 The Phules shared with Shinde an awareness of the separate axes of oppression that constitute
gender and caste, as well as the intermeshed nature of the two.

83. Consider the following statements:


1. SambadKaumudi was a Persian journal run by Raja Ram Mohan Roy.
2. RastGoftar was a Gujarati journal run by DadabhaiNaoroji.
Which of the statements given above is/are correct?

(a) 1 only
(b) 2 only
(c) Both 1 and 2
(d) Neither 1 nor 2
EXPLANATION:

Raja Ram Mohan Roy started two newspapers- one in Bangali, called the Sambad Kaumudi
and another in Persian, called the Mirat-ul-Akbar. Sambad Kaumudi was a noted pro-
Reformist publication that actively campaigned for the abolition of the Sati Pratha. So,
statement 1 is not correct.

Rast Goftar ("The Truth Teller") was an Anglo-Gujarati paper operating in Bombay that was
started in 1854 by Dadabhai Naoroji and Kharshedji Cama and championed social reform
among Parsis in Western India. So, statement 2 is correct.

ADDITIONAL INFORMATION:

Parsi Reform Movements

 The Rahnumai Mazdayasnan Sabha (Religious Reform Association) was founded in 1851 by a
group of English educated Parsis for the “regeneration of the social conditions of the Parsis and
the restoration of the Zoroastrian religion to its pristine purity”.

86
A SERIES

 The movement had Naoroji Furdonji, Dadabhai Naoroji, K.R. Cama and S.S. Bengalee as its
leaders.
 The message of reform was spread by the newspaper Rast Goftar (Truth-Teller). Parsi religious
rituals and practices were reformed and the Parsi creed redefined.
 In the social sphere, attempts were made to uplift the status of Parsi women through removal
of the purdah system, raising the age of marriage and education. Gradually, the Parsis emerged
as the most westernised section of the Indian society.
Raja Ram Mohan Roy

 Raja Ram Mohan Roy’s first published work Tuhfat-ul-Muwahhiddin (a gift to deists) published
in 1803 exposed irrational religious beliefs and corrupt practices of the Hindus as the belief in
revelations, prophets, miracles etc.
 In 1814, he founded Atmiya Sabha in Calcutta to campaign against idolatry, caste rigidities,
meaningless rituals and other social ills.
 He criticized the ritualism of Christianity and rejected Christ as the incarnation of God. In
Precepts of Jesus (1820), he tried to separate the moral and philosophical message of the New
Testament, which he praised, from its miracle stories.
 He founded the Atmiya Sabha in 1815, the Calcutta Unitarian Association in 1821, and the
Brahmo Sabha in 1828 which later became the Brahmo Samaj.
 He supported David Hare’s efforts to find the Hindu College in 1817, while Roy’s English school
taught mechanics and Voltaire’s philosophy.
 In 1825, he established Vedanta college where courses in both Indian learning and Western
social and physical sciences were offered.
 Ram Mohan found three journals- The Brahmanical Magazine (1821); The Bengali weekly,
Samvad Kaumudi (1821); and the Persian weekly, Mirat-ul-Akbar.

84. Who among the following is associated with the quinquennial settlement?
(a) Warren Hastings
(b) Alexander Reed
(c) Lord Cornwallis
(d) Thomas Munro
EXPLANATION:

In 1772 Warren Hastings made a 5 year settlement (The Quinquennial Settlement) of land
revenue. The objects of the experiment were to increase land revenue and collect it punctually.
Quinquennial Settlement (1772-1777) five-year land settlement replacing the traditional
zamindars with the highest bidding revenue speculators. Indian Diwans replaced the private
trade involved and corrupt collectors in the districts. The drawback of the settlement was, the
land had been over assessed and the state demand fixed very high. The result was that many
revenue collectors fell in heavy arrears. So, option (a) is correct.

The Ryotwari system was Introduced by Sir Thomas Munro and Captain Alexander Reed in
1820. In Ryotwari System the ownership rights were handed over to the peasants. British
government collected taxes directly from the peasants. The revenue rates of Ryotwari System

87
A SERIES

were 50% where the lands were dry and 60% in irrigated land. So, option (b) and (d) are not
correct.

Lord Cornwallis introduced a new revenue system under the Permanent Settlement of Bengal
in 1793 with a view to stabilize land revenue and create a loyal contented class of Zamindars.
This abolished periodic auction of Zamindari rights and established permanent Zamindari
rights to collect land revenue from the tenants and payment of a fixed amount to the
Government treasury every year. So, option (c) is not correct.

ADDITIONAL INFORMATION:

Significant Events under Warren Hastings 1773-1785

 Regulating Act of 1773.


 Act of 1781, under which the powers of jurisdiction between the governor-general-in-council
and the Supreme Court at Calcutta, were clearly divided.
 Pitt’s India Act of 1784.
 The Rohilla War of 1774.
 The First Maratha War in 1775-82 and the Treaty of Salbai in 1782.
 Second Mysore War in 1780-84.
 Strained relationships with Chait Singh, the Maharaja of Benaras, which led to Hastings’
subsequent impeachment in England.
 Foundation of the Asiatic Society of Bengal (1784).
85. “Gormati art” belongs to which of the following communities?
(a) Banjara community
(b) Gond community
(c) Bhil community
(d) Abhor community
EXPLANATION:

On the occasion of International Women's Day, Prime Minister shared a story of Maharashtra's
Beed district's Banjara community woman entrepreneur Vijaya Pawar. The video showcased
her inspiring journey of how she became an entrepreneur of age-old Gormati handicrafts of
the Banjara community. So, option (a) is correct.

ADDITIONAL INFORMATION:

Banjara community

 They are a Nomadic Tribe also known as Laman, Lambadi, and Banjari.
88
A SERIES

 They have a origin in the Marwar region of Rajasthan and distributed across the south Indian
states of Andhra Pradesh, Telangana, Karnataka and Maharashtra.
 They traditionally speak Lambadi / Gor Boli langauge.
 This includes performance arts such as dance and music as well as folk and plastic arts such
as rangoli, textile embroidery, tattooing and painting.
 The name of their festival is Teej during the month of August – a festival where the unmarried
Banjara girls pray for a good groom.
 The traditional dance forms of the Banjaras – Fire dance and Chari dance.
 This community believes in Hinduism.

86. Consider the following statements:


1. A Farmer Producer Organisation (FPO) is a legal entity registered under the Companies Act,
1956.
2. A Producer Organisation can be formed for non-farm activities too.
3. Producer organisations are also involved in quality control and export of the produce.
Which of the statements given above is/are correct?

(a) 1 and 2 only


(b) 2 and 3 only
(c) 1 and 3 only
(d) 1, 2 and 3
EXPLANATION:

A Producer Organisation (PO) is a legal entity formed by primary producers, viz. farmers, milk
producers, fishermen, weavers, rural artisans, craftsmen. artisan products, etc. it is a legal
entity registered under the Companies Act, 1956. So, statement 1 is correct.

A Producer Organisation (PO) is a generic name for an organization of producers of any


produce, e.g., agricultural, non-farm products. A PO can be a production company, a
cooperative society or any other legal form which provides for sharing of profits/benefits among
the members. In some forms like producer companies, institutions of primary producers can
also become a member of PO. Hence, it can be formed for non-farm activities too. So,
statement 2 is correct.

The concept of 'Farmer Producer Organizations, (FPO)' consists of collectivization of producers


especially small and marginal farmers so as to form an effective alliance to collectively address
many challenges of agriculture such as improved access to investment, technology, inputs,
and markets. FPO is one type of PO where the members are farmers. Producer organisations
can also play a role in governance, quality control and export of the produce. So, statement
3 is correct.

ADDITIONAL INFORMATION:

 Small Farmers’ Agribusiness Consortium (SFAC) was established in 1994 under Societies
Registration Act, 1860 as an autonomous body promoted by the Ministry of Agriculture &
Farmers' Welfare.

89
A SERIES

 Objectives of SFAc is by promoting agribusiness by encouraging institutional and private sector


investments and linkages to ensure the empowerment of all farmers in the country.
 Organising small and marginal farmers as Farmer Interest Groups, Farmer Producer
Organisations and Farmer Producer Company for endowing them with bargaining power and
economies of scale.
 Schemes Implemented by SFAC are Equity Grant & Credit Guarantee Fund (EGCGF) Scheme
, Venture Capital Assistance (VCA) Scheme, Farmer Producer Organization (FPO) Scheme,
National Agriculture Market (NAM) Scheme, etc.
 SFAC launched the Kisan Rath app with the help of officials of the Ministry of Agriculture
which lessened the problem of transport of farm produce during lockdown.
 It signed a Memorandum of Understanding (MoU) with the Agricultural and Processed Food
Products Export Development Authority (APEDA) to bring in better synergy in the agricultural
activities.

87. The ‘Priority Watch List’ that is frequently in news, keeps a watch on which of the following?
(a) Black money and terrorism funding
(b) Compliance of WTO guidelines
(c) Adherence to Intellectual Property Rights protection
(d) Development of nuclear weapons
EXPLANATION:

India continues to be on the ‘Priority Watch List’ of the United States Trade Representative
(USTR) for lack of adequate Intellectual Property (IP) rights protection and enforcement,
according to the USTRs Annual Special 301 Report. So, option (c) is correct.

ADDITIONAL INFORMATION:

Special 301 Report

 The Special 301 Report identifies trading partners that do not adequately or effectively protect
and enforce Intellectual Property (IP) rights or otherwise deny market access to U.S. innovators
and creators that rely on protection of their IP rights.
 The report is released annually by the United States Trade Representative (USTR).
 Trading partners that currently present the most significant concerns regarding IP rights are
placed on the Priority Watch List or Watch List. USTR identified 33 countries for these lists in
the Special 301 Report
 Algeria, Argentina, Chile, China, India, Indonesia, Russia, Saudi Arabia, Ukraine and
Venezuela are on the Priority Watch List.
 Barbados, Bolivia, Brazil, Canada, Colombia, Dominican Republic, Ecuador, Egypt,
Guatemala, Kuwait, Lebanon, Mexico, Pakistan, Paraguay, Peru, Romania, Thailand, Trinidad
& Tobago, Turkey, Turkmenistan, the United Arab Emirates, Uzbekistan and Vietnam are on
the Watch List.

88. With reference to dark matter and dark energy, which of the following statements is/are
correct?

90
A SERIES

1. They both account for the minority of the total content of the universe.
2. Dark matter does not interact with electromagnetic radiation such as light.
3. They are known to interact with the rest of the Universe through gravity only.
Select the correct answer using the code given below.
(a) 1 and 2 only
(b) 2 only
(c) 2 and 3 only
(d) 1 and 3 only
EXPLANATION:

Dark matter is a mysterious substance thought to compose perhaps about 27% of the makeup
of the universe. Dark energy makes up some 68% of the universe. So, statement 1 is not
correct.

Dark matter is invisible; it doesn’t emit, reflect or absorb light or any type of electromagnetic
radiation such as X-rays or radio waves. Thus, dark matter is undetectable directly. So,
statement 2 is correct.

Dark matter does interact with ordinary matter. It exhibits measurable gravitational effects on
large structures in the universe such as galaxies and galaxy clusters. Because of this,
astronomers are able to make maps of the distribution of dark matter in the universe, even
though they cannot see it directly. So, statement 3 is correct.

ADDITIONAL INFORMATION:

 The visible universe—including Earth, the sun, other stars, and galaxies—is made of protons,
neutrons, and electrons bundled together into atoms. Perhaps one of the most surprising
discoveries of the 20th century was that this ordinary, or baryonic, matter makes up less than
5 percent of the mass of the universe.
 The rest of the universe appears to be made of a mysterious, invisible substance called dark
matter (25 percent) and a force that repels gravity known as dark energy (70 percent).
 Dark matter is invisible; it doesn’t emit, reflect or absorb light or any type of electromagnetic
radiation such as X-rays or radio waves. Thus, dark matter is undetectable directly, as all of
our observations of the universe, apart from the detection of gravitational waves, involve
capturing electromagnetic radiation in our telescopes.
 Yet dark matter does interact with ordinary matter. It exhibits measurable gravitational effects
on large structures in the universe such as galaxies and galaxy clusters. Because of this,
astronomers are able to make maps of the distribution of dark matter in the universe, even
though they cannot see it directly.
 They do this by measuring the effect dark matter has on ordinary matter, through gravity.

89. Consider the following statements:


1. Gravitational waves are associated with electromagnetic radiation released by non
accelerating objects.

91
A SERIES

2. Gravitational lensing is the phenomenon of bending of light around a massive object like
black hole.
3. Both phenomena are explained by Einstein special theory of relativity.
Which of the statements given above is/are correct?
(a) 2 only
(b) 1 and 2 only
(c) 2 and 3 only
(d) 1 and 3 only
EXPLANATION:

Gravitational waves are disturbances in the curvature of spacetime, generated by accelerated


masses, that propagate as waves outward from their source at the speed of light. Gravitational
waves transport energy as gravitational radiation, a form of radiant energy similar to
electromagnetic radiation. So, statement 1 is not correct.

A gravitational lens is a distribution of matter (such as a cluster of galaxies) between a distant


light source and an observer, that is capable of bending the light from the source as the light
travels towards the observer. This effect is known as gravitational lensing. So, statement 2 is
correct.

General relativity, also known as the general theory of relativity, is the geometric theory of
gravitation published by Albert Einstein in 1915 and is the current description of gravitation
in modern physics. Gravitational waves and Gravitational lensing are explained by Einstein
General theory of relativity not special theory of relativity. So, statement 3 is not correct.

ADDITIONAL INFORMATION:

Gravitational waves

 Gravitational waves are invisible ripples that form when a star explodes in a supernova; when
two big stars orbit each other; and when two black holes merge.
 Travelling at the speed of light, gravitational waves squeeze and stretch anything in their path.
 Detecting gravitational waves
 Gravitational waves were proposed by Albert Einstein in his General Theory of Relativity over
a century ago.
 It was only in 2015, however, that the first gravitational wave was actually detected — by LIGO.
Since then, there have been a number of subsequent detections of gravitational waves.
 The signal detected at LIGO and Virgo, as described by the LIGO Collaboration, resembled
“about four short wiggles” and lasted less than one-tenth of a second.

Gravitational Lensing

92
A SERIES

 The phenomenon occurs when a huge amount of matter, such as a massive galaxy or cluster
of galaxies, creates a gravitational field that distorts and magnifies the light from objects behind
it, but in the same line of sight.
 In effect, these are natural, cosmic telescopes; they are called gravitational lenses.
 These large celestial objects will magnify the light from distant galaxies that are at or near the
peak of star formation.
 The effect allows researchers to study the details of early galaxies too far away to be seen
otherwise with even the most powerful space telescopes.

90. Consider the following statements with respect to East India company’s trade and influence
in India in initial years:
1. The East India Company was initially a mere petitioner who wanted trade concessions from
Mughal rulers for their commercial interests.
2. North India was easier for the company to expand their control than the south and west.
Which of the statements given above is/are correct?

(a) 1 only
(b) 2 only
(c) Both 1 and 2
(d) Neither 1 nor 2

91. Which of the following can be considered as the most suitable argument(s) to justify the
British conquest of India?
1. The British came merely as traders but were drawn into the political turmoil created by Indian
rulers and were thus forced to acquire territories.
2. They came with a clear blue print to colonize India by working on it bit by bit over the years.
3. Initially they came only with commercial interest but gradually developed an imperialistic
mindset to exploit the resources of India.
4. They were invited by some sections of Mughal nobility to defeat the Mughal emperor and
establish their own independent states.
Select the correct answer using the code given below.

(a) 3 only
(b) 1 and 3 only
(c) 1, 2 and 3 only
(d) 4 only

92. “Europa”, “Ganymede”, “Callisto”,are the names of the moons of which of the following
planets of the solar system?
(a) Jupiter
(b) Saturn
(c) Uranus
(d) Neptune
EXPLANATION:

93
A SERIES

There are 79 known moons of Jupiter, not counting a number of moonlets likely shed from the
inner moons. The most massive of the moons are the four Galilean moons, which were
independently discovered in 1610 by Galileo Galilei and Simon Marius and were the first
objects found to orbit a body that was neither Earth nor the Sun. They are the four largest
moons of Jupiter—Io, Europa, Ganymede, and Callisto. So, option (a) is correct.

ADDITIONAL INFORMATION:

Asteroids, Meteoroids and Comets

 Asteroids or Planetoids are rocky bodies up to 800 km in diameter, although most are much
smaller in diameter less than a km, i.e. Asteroids are the minor planets which especially belong
to the inner solar system.
 They orbit the Sun in the asteroid belt, which lies between the orbits of Mars and Jupiter.
 Meteoroids are small fragments of rock and metal travelling through the space.
 Upon reaching the Earth's surface they are called Meteorites.
 Comets: They are the smallest units of the cosmic bodies which is made up of frozen gases,
rocks and dusts.
 The tail of the comet always points away from the Sun because of the force exerted by solar
wind and the radiation pressure.The most common comet is Halley’s comet which is spotted
every 76 years.

93. Consider the following statements with respect to ‘drain of wealth’ theory propounded by
early Indian nationalists:
1. Drain was the portion of national product of India which was not available to Indians but was
taken away by Britishers.
2. It was first put forward by Dadabhai Naoroji.
Which of the statements given above is/are correct?

(a) 1 only
(b) 2 only
(c) Both 1 and 2
(d) Neither 1 nor 2

94. Which of the following was the impact of reforms introduced by British in Indian Judiciary?
1. It established the rule of law by replacing personal law of rulers with a codified law.
2. It eased the access to judiciary for Indians.
3. It brought European subjects under the jurisdiction of written laws.
4. Government servants were now made answerable to civil courts.
Which of the statements given above is/are correct?

(a) 1 and 2 only


94
A SERIES

(b) 2, 3 and 4 only


(c) 3 and 4 only
(d) 1, 3 and 4 only

95. With respect to the concept of Quantum computing, recently in news, which of the following
statements is/are correct?
1. In quantum computing, a bit is a single piece of information that can exist in two states – 1 or
0.
2. In quantum computing, a qubit is a unit of quantum information.
Select the correct answer using the codes given below

(a) 1 only
(b) 2 only
(c) Both 1 and 2
(d) Neither 1 Nor 2

96. Consider the following statements regarding the NavIC technology.


1. Navic is the operational name of the Indian Regional Navigation Satellite System (IRNSS)
developed by ISRO.
2. It is designed to provide accurate position information service to users in India as well as the
region extending up to 1500 km from its boundary.
3. Navic is expected to provide a position accuracy of better than 20 m in the primary service
area.
Which of the statements given above is/are correct?

(a) 1 only
(b) 1 and 2 only
(c) 2 and 3 only
(d) 1, 2 and 3

97. The Generalised System of Preferences


1. Provides duty-free treatment to goods of designated beneficiary countries.
2. Is the concept same as that of Most FavouredNation.
3. Is instituted under the aegis of UNCTAD.
Select the correct answer using the codes given below

(a) 1 and 2 only


(b) 1 and 3 only
(c) 2 and 3 only
(d) 1, 2 and 3

98. Consider the following statements regarding the Genome India Project.

95
A SERIES

1. Its aim is to ultimately build a grid of the Indian “reference genome”, to fully understand the
type and nature of diseases and traits that comprise the diverse Indian population.
2. It hopes to form a grid after collecting 10,000 samples in the first phase from across India, to
arrive at a representative Indian genome.
3. It will help to achieve the deliberate transfer of DNA for therapeutic purposes.
Which of the statements given above is/are correct?

(a) 1 only
(b) 1 and 2 only
(c) 2 and 3 only
(d) 1, 2 and 3

99. Which one of the following statements regarding the Blue Dot Network, often seen in News,
is correct?
(a) It is a China led initiative for the infrastructure development.
(b) It is a BRICS’ initiative for financing loans for developmental projects.
(c) It is a new kind of parcel service initiated by Australia.
(d) It is a multi-stakeholder initiative to provide assessment and certification of
infrastructure development projects.

100. The 15th Finance Commission, its interim report for 2020/21, has recommended a one
percentage point reduction in the vertical split of the divisible pool of tax revenues accruing
to States to 41% from the 42%. What may be the probable reason for this reduction?
(a) Due to Poor economic Growth.
(b) Due to Corona Pandemic and lockdown impact
(c) Due to the strict adherence to the FRBM Act, 2003.
(d) Due to Reorganisation of Jammu and Kashmir into Union Territory.

96

You might also like